Ethical issues in psychology MCQs PDF

Key Points

 The ethical considerations in research:

Ethical considerations in research are a set of principles that guide your research designs and practices. Scientists and researchers must always adhere to a certain code of conduct when collecting data from people. It includes:

Voluntary participation- Voluntary participation means that all research subjects are free to choose to participate without any pressure or coercion.

Informed consent- Informed consent refers to a situation in which all potential participants receive and understand all the information they need to decide whether they want to participate. This includes information about the study’s benefits, risks, funding, and institutional approval.

Anonymity- Anonymity means that the researcher does not know who the participants are and they can’t link any individual participant to their data.

Confidentiality- Confidentiality means that the researcher know who the participants are, but they remove all identifying information from the report.

Potential for harm- It means that physical, social, psychological and all other types of harm are kept to an absolute minimum.

Results communication- The way the researcher communicates the research results can sometimes involve ethical issues. Good science communication is honest, reliable, and credible. It’s best to make the results as transparent as possible. It includes avoiding Plagiarism. Plagiarism means submitting others’ works as your own. Although it can be unintentional, copying someone else’s work without proper credit amounts to stealing. It’s an ethical problem in research communication because they may benefit by harming other researchers.

Hence, The ethical considerations in research includes:

(B) Informed consent

(C) Avoiding plagiarism
(D) Proper referencing

You are a psychologist treating a 20 year old college student for Sx's of Depression. The student is covered by her parents' health insurance and has given you permission to speak only with the insurance company. The student's mother calls you to inquire about her child's progress in Tx. What should you do?

A. Provide supportive therapy to the mother and schedule a family therapy session.

B. Deny that you are meeting with the student for therapy and hang up.

C. Confirm that you are seeing the student but refuse to share any information about your sessions.

D. Neither confirm nor deny that you are seeing the student for treatment.

D. Neither confirm nor deny that you are seeing the student for treatment. As long as the student has not given you specific consent to speak with her mother, you should neither confirm nor deny that you are seeing the student for therapy. Even thought the student is covered by her parents' insurance, you do not have legal permission to speak with them.

You receive a subpoena requiring you to testify about a current client at a trial. You contact the client who says he does not want you to testify. You should:

A. Notify the court that you will not be appearing at the trial because the client has not given you permission to do so

B. Appear at the trial as requested but claim the privilege on behalf of the client

C. Explain to the client that you are legally required to provide the requested information

D. Do nothing until you receive an order from the court

B. Appear at the trial as requested but claim the privilege on behalf of the client A subpoena is valid even when a client does not give permission for the release of confidential information. However, in this situation, you should contact the client’s attorney or your own attorney to discuss the matter or contact the attorney who issued the subpoena to request to be released from it. Unless you receive the requested release, you must appear as requested but should claim the privilege on the client’s behalf. (AATBS Facebook Question #61)

If you believe that your adult client is gravely disabled, you:

A. May take the steps necessary to have the client involuntarily hospitalized if his grave disability is due to a mental disorder

B. Must have permission from the client’s family to have the client hospitalized unless the client poses a danger to others

C. Should take the steps necessary to have the client involuntarily hospitalized only if he also presents a danger to others

D. Cannot involuntarily hospitalize the client unless his grave disability is due to chronic alcoholism or substance abuse

A. May take the steps necessary to have the client involuntarily hospitalized if his grave disability is due to a mental disorder When a client’s grave disability is due to a mental disorder or chronic alcoholism or drug use, you may take the steps necessary to have the client involuntarily hospitalized as provided in WIC Sections 5150 and 5230. (AATBS Facebook Question #60)

A psychologist who obtained a Ph.D. in experimental psychology wants to change her specialty to clinical psychology. To meet the requirements set forth by the General Guidelines for Providers, the psychologist must

A. complete an internship in clinical psychology under the supervision of a professional clinical psychologist.

B. complete appropriate doctoral-level classes and supervised post-doctoral training.

C. obtain a Ph.D. in clinical psychology from an accredited college or university.

D. meet her state's requirements for licensure in clinical psychology.

B. complete appropriate doctoral-level classes and supervised post-doctoral training. The licensing exam often contains questions such as these, and in approaching them, you should remember these buzzwords: training AND experience. Specifically, the Specialty Guidelines state that "professional psychologists who wish to qualify as clinical psychologists meet the same requirements with respect to subject matter and professional skills that apply to doctoral and postdoctoral education and training in clinical psychology." Choice "B" is the only one that includes an element of both training and experience, and is therefore the best answer. Moreover, unlike as stated by choice "C", a second Ph.D. would not be necessary -- doctoral level coursework in clinical psychology would be sufficient to meet the academic training aspect of this requirement

Continuing Education Credits (CE Credits) earned through an APA approved sponsor means that the program is:

A. sanctioned by APA

B. endorsed by APA.

C. approved by APA.

D. the responsibility of the sponsor.

D. the responsibility of the sponsor. An organization is approved by the APA to sponsor continuing education programs. The sponsor then becomes responsible for each program. The APA periodically asks for reports from the sponsor, but the specific program is not endorsed, sanctioned, or approved by the APA. Only the overall sponsorship is approved by the APA.

While attending a staff meeting at a mental health facility where you work, you learn that one of your colleagues, a psychologist, is behaving in an unethical manner toward one of her patients. Despite being advised to do so by several members of the staff, she states that she will not change her behavior. What should you do in this situation?

A. Approach the colleague individually and urge her to change her behavior; if she refuses, report her to the appropriate ethics committee.

B. Approach the colleague individually and urge her to change her behavior; if she refuses, recommend to the director of the facility that she be fired.

C. Report the colleague to the appropriate ethics committee immediately, since she has already stated that she will not change her behavior.

D. Announce in the staff meeting that you want to "go on the record" as noting that the psychologist is behaving unethically.

A. Approach the colleague individually and urge her to change her behavior; if she refuses, report her to the appropriate ethics committee. The Ethics Code states that you should deal with ethical violations by another psychologist in an informal manner, by bringing it to the attention of the psychologist. A report to an ethics committee should be made when the attempt at informal resolution has failed or if the violation is not amenable to informal resolution. Thus, you should speak to the psychologist privately and report her to an ethics committee if she still refuses to change her behavior. You may have thought that, since the psychologist has already stated her refusal to behave ethically, you should go ahead and report her, without bothering to talk to her. This actually makes sense, but the Ethics Code requires that you attempt an informal attempt at resolution first. A staff meeting is a formal setting, and does not personally give you the opportunity to attempt an informal resolution.

You have been working with a couple in marital therapy for four months, with slow but notable progress. After a session one day, the wife pulls you aside and asks to make some individual appointments with you. You should

A. refer her to another therapist.

B .wait until the completion of marital therapy and then see her individually.

C. terminate with the couple and see the woman individually.

D. encourage her to discuss any relevant issues within the context of marital therapy.

A. refer her to another therapist. There is a potential for role conflict when a client in conjoint therapy asks you to see him or her individually. It's possible that there may be a conflict between your considerations as a couple's therapist and as an individual therapist. Thus, although your actions would vary depending on the parameters of the individual case, the best answer in light of the information we have is to refer the woman to another therapist for individual therapy.

Fee arrangements should be made with clients

A. at the beginning of treatment.

B. at the end of the first session.

C. on the phone before the first session.

D. as soon as possible.

D. as soon as possible. This is an example of a question with more than one good answer. Choices “A”, “B”, and “C” may all be acceptable, depending on the circumstances. But because there are a number of acceptable answers to this question, you'd hope to find one general answer, such as “D”, that covers most or all of the specific possible options. More importantly, of the choices listed, “D” is most consistent with the language of APA's Ethical Standard 6.04(a), which states: “As early as is feasible in a professional or scientific relationship, psychologists and recipients of psychological services reach an agreement specifying compensation and billing arrangements.

A client suspects that she may have been sexually abused as a child, although she has no conscious recollection of the abuse. She asks her psychologist to use hypnosis to help her retrieve any repressed memories she may have of any abuse. The psychologist should:

A. agree to use hypnosis only if he or she has obtained adequate training and experience in it's use.

B. agree to use hypnosis but take detailed notes in the event of future legal action and avoid asking the client any leading questions.

C. advise against the use hypnosis, but recommend the use of guided imagery, which may be more admissible in court.

D. advise the patient that hypnosis may produce false recollections of abuse and is therefore inappropriate.

D. advise the patient that hypnosis may produce false recollections of abuse and is therefore inappropriate. In a report titled "Final Conclusions of the American Psychological Association Working Group on Investigation of Memories of Childhood Abuse" [Psychology, Public Policy, and Law, 2000, 4 (4), 933-940] the authors acknowledge that "it is possible for memories of abuse that have been forgotten for a long time to be remembered." However, they recommend that "clients who seek hypnosis as a means of retrieving or confirming their recollections should be advised that it is not an appropriate procedure for this goal because of the serious risk that pseudomemories may be created in trance states and of the related risk due to increased confidence in those memories. Clients should also be informed that the use of hypnosis could jeopardize any future legal actions they might want to take."

An attorney contacts a psychologist requesting that his client be given a psychological evaluation prior to appearing in court. Following the evaluation the client asks that his records not be released to the court, although his attorney believes that releasing the records would be in his client's best interest. The psychologist should:

A. release the records since there is no privilege regarding forensic evaluations

B. release the records since the attorney requested the evaluation

C. not release the records

D. seek consultation

C. not release the records. Since this does not appear to be a court-ordered evaluation, the examinee remains the holder of the privilege; thus, his records should not be released to the court without his consent.

The purpose of State Licensing Boards is primarily to

A. educate the public.

B. educate the profession.

C. protect the public.

D. set minimum standards of practice.

C. protect the public. The State Boards derive their authority from the principle of protecting the public. That's the justification for giving exams, setting requirements for licensure, writing and enforcing regulations, and so on.

You are working with a couple in marital therapy and are conducting the initial interview. You realize that although the husband doesn't remember you, the two of you once dated. You should

A. speak to the husband alone, explain the situation and ask him if he feels comfortable with proceeding.

B. see the wife in individual therapy only.

C. refer the couple to another therapist.

D. let the couple know the situation and then proceed with therapy.

C. refer the couple to another therapist. This question is fairly easy to answer once you remember you are not obliged to provide services for all your referrals. This is an initial interview and you have a situation that involves a multiple relationship. You need to refer this couple to one of your competent colleagues.

A manufacturing company hires a psychologist to screen job applicants using standardized cognitive ability tests. The company then asks the psychologist to train their human resources staff to administer and score the tests. The psychologist should:

A. agree to do so if the psychologist is able to provide adequate training to the staff

B. agree to do so if the psychologist is able to supervise the human resources staff

C. refuse to do so because cognitive ability tests are not valid predictors of job performance

D. refuse to do so because the human resources staff lacks the appropriate qualifications

D. refuse to do so because the human resources staff lacks the appropriate qualifications Ethical Standard 9.07 states, "Psychologists do not promote the use of psychological assessment techniques by unqualified persons." A human resources staff would be unqualified to use these tests and should not be trained in their use. Contrary to C, cognitive ability tests are considered to be a relatively good predictor of job performance.

Utilization review, an important component of managed health care, refers to the idea that it is useful to

A. review benefits to eliminate or reduce unnecessary health care resources.

B. determine the adequacy of health care standards by comparing them to predetermined standards.

C. make decisions on patient care by a team of medical experts rather than an individual physician.

D. allow a patient to choose from several insurance plans.

A. review benefits to eliminate or reduce unnecessary health care resources. Utilization review is concerned with conserving health care monies. It does this through having a utilization review committee assess the use of benefits and reduce or eliminate inappropriate or unnecessary use of health care resources. Answer B is a description of the concept of quality assurance and answer C is describing a medical team management approach to individual health care.

The Health Insurance Portability and Accountability Act (HIPAA) of 1996 applies to:

A. all health care providers

B. only health care providers who transmit patient information by fax and the internet

C. only health care providers who employ someone, such as billing services, with access to patient information

D. only health care providers who transmit patient information electronically

D. only health care providers who transmit patient information electronically HIPAA does not apply to all health care providers. It only applies to “cover entities” — health plans, health care clearinghouses and healthcare providers — who transmit health information electronically. A provider who only communicates information by mail, phone, or fax is not a covered entity. Transmitting information electronically refers to use of the internet and includes receiving health information as well as transmitting information. For example, a provider is a covered entity even if she/he never transmits information but receives information, such as a description of benefits e-mail, via the internet. Additionally, a provider employing someone who sends or receives patient information on the provider’s behalf via the internet is also a covered entity.

A college psychology instructor gives a battery of psychological tests to some of his students who volunteered to be participants. Based on these tests, he finds that one of the participants is at risk for emotional disturbance. He talks to this student and suggests counseling. At the same time, he informs the director of the college counseling office who is a friend of his. According to APA ethical principles, the professor acted

A. unethically, because he revealed this information to a third party.

B. ethically, because he acted in accord with the student's welfare.

C. ethically, because he had responsibilities to the college as well as to the student.

D. unethically, because he isn't a clinician.

A. unethically, because he revealed this information to a third party. Suggesting counseling to the student is ethical and appropriate. However, barring an emergency situation characterized by imminent danger, informing a third party is unethical.

A psychologist is undertaking a research program in an elementary school on the effect of a new counseling program on enhancing children's self-esteem. The psychologist has received permission from the school's principal, the teachers, and from the parents of each student. However, when she explained the program to the children, some of them objected to being part of the study. The psychologist should

A. not use these children in her study.

B. call the parents of these children and ask them to try to convince the children to participate.

C. use the children in her study because she has all necessary legal consents.

D. re-design the study to eliminate the objection the children have to participating.

A. not use these children in her study. The best idea is to just leave out these children. Participants have the right to decline to participate in research, even if they are children. It wouldn't be practical or prudent to ask her to re-design the whole experiment because a handful of children object to being participants. Thus, the best answer is to just do the study without using the children who object to participating.

You have a client who does not want their records released under any circumstances. The attorney has subpoened you. Your best course of action is to:

A. Respond in-person, but don't take the records

B. Take the records, but don't turn them over

C. Ignore the entire matter

D. Destroy the records

B. Take the records, but don't turn them over You should respond to the subpoena by asserting the psychotherapist-client privilege on the client's behalf. If you cannot be released from the subpoena, you should appear at the legal proceeding with the documents requested, however, you should not actually release any information in the records, unless the judge orders you to do so. Keep in mind that a subpoena duces tecum requires a person to appear at a designated time and place with a copy of the records. It does not necessarily require the person to release those records.

A 16-year-old girl tells her therapist that she wants to kill herself and that she's been thinking of using her mother's prescription pain pills to overdose. The therapist then tells her that he will have to tell her parents about her suicide risk. The girl becomes enraged and says that she would not have disclosed the information if she knew that the therapist would tell her parents. The therapist should:

A. agree not to tell the parents if the girl agrees to a no-harm contract

B. agree not to tell the parents, but tell them without the girl's knowledge

C. tell the parents and refer the girl to another therapist because she will probably not be able to trust the therapist again

D. tell the parents and apologize to the girl for having to do so

D. tell the parents and apologize to the girl for having to do so According to both ethics and law, a psychologist may disclose confidential information without the client's consent to protect the client from harm. This applies to minors and adults alike. In this case, it would most likely be appropriate to inform the girl's parents about her suicide risk. The reasons for this should be explained to the girl and the therapist should apologize for the breach of confidentiality. Given the apparent level of risk involved, it would probably not be sufficient to rely on a no-harm contract (A). It would certainly be inappropriate and countertherapeutic to lie to the client (B). And it may not be necessary to refer the client (C). If handled empathically, it is likely that the girl will come to understand the therapist's reasons for disclosing to the parents and may come to appreciate that her safety was his overriding concern.

Miranda M.,‭ ‬age‭ ‬32,‭ ‬makes an appointment with you at the recommendation of her physician who has diagnosed her recurrent headaches as the result of stress.‭ ‬The woman tells you that a friend of hers was trained in self-hypnosis for tension headaches and says that she also wants training in self-hypnosis.‭ ‬You took a weekend workshop on hypnosis in graduate school but have not used it in your clinical practice.‭ ‬As an ethical psychologist,‭ ‬you should:‬‬‬‬‬‬‬‬‬‬‬‬‬‬

a. Admit your lack of expertise in the use of self-hypnosis and refer the woman to another therapist.

b. Advise the woman that the underlying cause of her headaches would probably not be dealt with by self-hypnosis and suggest that she begin individual psychotherapy.

c. Accept the woman for treatment and begin training her in self-hypnosis after reviewing your workshop notes.

d. Accept the woman for treatment and begin training her in self-hypnosis but obtain consultation if you encounter any problems during the course of treatment.

a. admit your lack of expertise in the use of self-hypnosis and refer the woman to another therapist. (Correct Answer)‬ Of the responses given, this one is most consistent with the requirement to provide services that are within the boundaries of your professional competence. Note that, while it is up to you to determine your competence with regard to a particular service or technique, a weekend workshop in self-hypnosis is clearly inadequate. Providing services within the boundaries of one’s competence is addressed by Standard 2.01 of the APA’s Ethic Code and Principle II.6 of the Canadian Code of Ethics. b. advise the woman that the underlying cause of her headaches would probably not be dealt with by self-hypnosis and suggest that she begin individual psychotherapy. - Incorrect It would be unethical to try to convince the client to choose another course of treatment simply because you are not trained in the treatment she desires. c. accept the woman for treatment and begin training her in self-hypnosis after reviewing your workshop notes. - Incorrect Reviewing your workshop notes would not be adequate training. d. accept the woman for treatment and begin training her in self-hypnosis but obtain consultation if you encounter any problems during the course of treatment. Incorrect Although this response comes closer to fulfilling the requirements of the ethical guidelines than do responses b and c it falls short of those requirements because it implies that consultation would not be sought unless you believe that you are having problems during the course of treatment. Consequently, response a is a better answer.

(AATBS Ethics Domain Quiz #18441)

An “informed consent” from a client:

A. Must always be in writing

B. Must always be in writing and in the client’s native language

C. May be either verbal or in writing

D. May be either verbal or in writing but must be documented

D. May be either verbal or in writing but must be documented (Correct) Standard 3.10 of the APA’s Ethics Code stated tht consent must be “appropriately documented,” and Principle I.22 of the Canadian Code of Ethics states that it is acceptable to “accept and document oral consents” when there is a good reason not to obtain a signed written consent. Therefore, this is the best answer of those give. The issue of informed consent is addressed in Standard 3.10 of the APA’s Ethics Code and Principle I.22 of the Canadian Code of Ethics.

(AATBS Ethics Domain Quiz #18434)

You receive a call from Mrs.‭ ‬Wang who is very upset because her‭ ‬14-year-old daughter witnessed the assault of her best friend three days ago and is very distraught. ‭ ‬The girl hasn’t slept,‭ ‬won’t eat,‭ ‬and can’t stop crying.‭ ‬You have limited experience working with adolescents and in providing crisis intervention services.‭ ‬However,‭ ‬there is no one else in the community who is more experienced than you are.‭ ‬As an ethical psychologist,‭ ‬you will:‬‬‬‬‬‬‬‬‬‬‬‬‬‬‬‬‬‬‬‬‬‬‬‬‬‬‬‬‬‬

a. agree to see Mrs.‭ ‬Wang’s daughter in therapy since you’ve had some experience providing crisis intervention experiences. ‬‬‬

b. inform Mrs.‭ ‬Wang about your lack of experience and let her decide if she wants you to provide therapy to her daughter. ‬‬‬

c. inform Mrs.‭ ‬Wang that you cannot see her daughter because of your lack of experience. ‬‬‬

d. see Mrs.‭ ‬Wang’s daughter in therapy only until the crisis has ended or until you locate alternative services.‬‬‬

d. see Mrs.‭ ‬Wang’s daughter in therapy only until the crisis has ended or until you locate alternative services. CORRECT This answer is most consistent with ethical requirements. Standard 2.02 of the Ethics Code, for example, states that “in emergencies, when psychologists provide services to individuals for whom other mental health services are not available and for which psychologists have not obtained the necessary training, psychologists may provide such services in order to ensure that services are not denied. The services are discontinued as soon as the emergency has ended or appropriate services are available.” Standard 2.02 of the APA’s Ethics Code and Principle II.8 of the Canadian Code of Ethics apply to this situation.

(AATBS Ethics Domain Quiz #18476)

If a psychologist acts as both a fact witness for the plaintiff and an expert witness for the court in a criminal trial,‭ ‬she has acted:‬‬

a. unethically by accepting dual roles.

b. ethically as long as she did not have a prior relationship with the plaintiff.

c. ethically as long as she clarifies her roles with all parties.

d. ethically as long as she obtains a waiver from the court.

c. ethically as long as she clarifies her roles with all parties. (Correct Answer) According to ethical guidelines, accepting multiple roles may be acceptable as long as certain conditions are met – e.g., as long as the psychologist clarifies the nature of the multiples relationships with all involved parties (see Standard 3.05 of the APA’s Ethics Code and Principle I.26 of the Canadian Code of Ethics). In addition, the Specialty Guidelines for Forensic Psychologists states that acting as a consultant for one party and a fact witness for another may be acceptable as long as the psychologist clarifies his/her roles with both parties and acts in a way that does not compromise his/her judgment and objectivity. The adoption of multiple roles is addressed in the ethics codes published by the American and Canadian Psychological Associations and in the APA’s Specialty Guidelines for Forensic Psychologists. (AATBS Ethics Domain Quiz #18560)

The mother of a‭ ‬31-year-old client that you have been seeing for six months calls to check on her son's progress.‭ ‬She says that she is very concerned about her son and wants to know if there is anything she can do to help him.‭ ‬As an ethical psychologist,‭ ‬you:‬‬‬‬‬‬‬‬‬‬‬‬

a. advise her in general terms about how she can help her son.

b. suggest that she make an appointment so you can discuss the matter in person.

c. suggest that she come to therapy with her son at his next appointment.

d. tell her that you cannot discuss her son’s condition with her.

d. tell her that you cannot discuss her son’s condition with her. (CORRECT) Giving the mother any information about her adult son would represent a breach of his confidentiality. Therefore, this is the best answer of those given. This question addresses the issue of client confidentiality.

(AATBS Ethics Domain Quiz #18510)

Sexual Harassment:

A. Is explicitly prohibited by ethical guidelines

B. Is not explicitly prohibited by ethical guidelines but is implicitly prohibited by standards limiting “multiple relationships.”

C. Is not explicitly prohibited by ethical guidelines but is implicitly prohibited by standards limiting “sexual intimacies.”

D. Is not Explicitly prohibited by ethical guidelines but is implicitly prohibited by standards requiring “avoidance of harm.”

A. Is explicitly prohibited by ethical guidelines (Correct) Standard 3.02 of the Ethics Code states that “psychologists do not engage in sexual harassment”; Principle I.4 of the Canadian Code of Ethics states that psychologists “abstain from harassment, including sexual harassment.” Sexual Harassment is explicitly addressed by Standard 3.02 of the APA’s Ethics Code and Principle I.4 of the Canadian Code of Ethics.

(AATBS Ethics Domain Quiz #18555)

Kameko K., a psychology intern, administers psychological tests to the clients of a community mental health clinic, but her supervisor always scores the tests and interprets their results. When preparing the psychological report for these clients, the supervisor does not indicted that the intern administered the tests. This is:

A. Ethical because the supervisor scored and interpreted the tests.

B. Ethical because the supervisor is responsible for the accuracy of the psychological report.

C. Unethical because an intern should not be administering psychological tests.

D. Unethical because the intern should be listed as the examiner in the report.

D. Unethical because the intern should be listed as the examiner in the report. (Correct) This response is most consistent with ethical guidelines. For example, Standard 5.01(b) of the APA’s Ethics Code prohibits psychologists from making misleading statements and Standard 6.06 requires psychologists to provide accurate information to payors and funding sources. It is also consistent with Principles III.1 and 5 of the Canadian Code of Ethics. This issue is not directly addressed by ethical guidelines, so the best approach when choosing the answer would have been to identify the one that is most consistent with the “spirit” of the guidelines.

(AATBS Ethics Domain Quiz #18470)

Of the following, which is responsible for the largest population of complaints filed with the APA's Ethics Committee?

A. Sexual Misconduct

B. Test Misuse

C. Breach of Confidentiality

D. Discrimination

A. Sexual Misconduct (Correct) The 2009 Report of the Ethics Committee, for example, identifies sexual misconduct of an adult as being the behavior most commonly cited in cases opened by the Ethics Committee, which is consistent with previous reports. Test misuse, breach of confidentiality, and discrimination (answer b, c, and d) are causes of complaints but not as often as is sexual misconduct. Although the types of actions that underlie complaints filed with the APA's Ethics Committee vary in number from year to year, one of the actions listed in the answers to this question has consistently been the most frequent cause of complaints.

(AATBS Ethics Domain Quiz #18523)

When evaluating a defendant’s competence to stand trial,‭ ‬the focus of the evaluation is on the defendant’s:

A. ability to distinguish right from wrong.

B. ability to comprehend the charges against him/her.

C. mental state at the time he/she committed the crime.

D. DSM-IV diagnosis.

B. ability to comprehend the charges against him/her - (CORRECT) An evaluation for competence to stand trial focuses on the defendant’s ability to comprehend the charges against him/her, to understand the trial process, to cooperate with his/her attorney, etc. The focus of an evaluation to determine a defendant’s competence to stand trial focuses on his/her current psycholegal abilities and impairments.

(AATBS Ethics Domain Quiz #18565)

You receive a letter from Mrs. Bisque's attorney asking you to testify that Mrs. Bisque, is in the process of the divorce settlement, should have sole custody of her children. You are aware that Mr. Bisque was recently admitted to the psychiatric hospital in another city. You should:

A. Make a recommendation based on your knowledge that Mr. Bisque is obviously unable to have custody of the children.

.B. Make a recommendation based on an interview with Mrs. Bisque and her children.

C. Refused to make a recommendation since psychologists are not supposed make specific custody recommendations.

D. Refused to make a specific recommendation about custody unless you can evaluate the entire family.

D. Refused to make a specific recommendation about custody unless you can evaluate the entire family (Correct) An accepted standard of practice in custody situations is that a psychologist should not make a recommendation for custody unless the psychologists have evaluated all parties. This is consistent with the requirements of standard 9.01 of the APA's Ethics Code and Principles II.3 of the Canadian Code of Ethics and with the provisions of the APA's guidelines for Child Custody Evaluation and Divorce Proceeding. Although it is not unethical to provide information (e.g., test results) about a single party in a custody dispute, it would be unethical to make a recommendation about custody based on incomplete information.

(AATBS Ethics Domain Quiz #18465)

A client that Dr. William Wong has been seeing in therapy for six months is currently the defendant in a court case. The client's attorney contacts Dr. Wong, requesting that he provide her with information about the client's diagnosis and treatment progress since this information will be helpful in preparing the client's defense. Dr. Wong should:

A. Release that information as requested since privilege is waived in the situation

B. Release only the information he believes is relevant to the client's case

C. Release the information since the request came from the client's attorney

D. Discussed the release with the client before taking any additional action

D. Discussed the release with the client before taking any additional action (Correct) Privilege may or may not be waived in the situation and, even if it is, Dr. Wong should discuss the release of confidential information with the client before doing so (and, in most cases, obtain a waiver from the client). The release of confidential client information ordinarily requires a waiver of confidentiality from the client.

(AATBS Ethics Domain Quiz #18471)

A psychologist realizes that one of her therapy client is a co-owner of the company that just hired her as a consultant. To be consistent with ethical requirements, the psychologist should:

A. Remedy the situation in a way that the client's best interests into account

B. Continue in both roles but discuss any problems when they occur

C. Terminate her role of the therapist or consultant

D. Terminate her role of consultant

A. Remedy the situation in a way that the client's best interests into account (Correct) the provisions of the APA and CPA ethical guidelines are somewhat ambiguous with regard to this issue. However, Standard 3.05 (b) of the APA's Ethics Code states that, within a multiple relationship occurs, "the psychologist takes reasonable steps to resolve it with due regard for the best interest of the affected person and maximal compliance with the Ethics Code"; and the Values Statement of Principle III of the Canadian Code of Ethics contains similar language. This is a difficult question to answer none of the answers is incorrect. However, the best answer is the one that comes closest to the actual language of the ethical guidelines.

(AATBS Ethics Domain Quiz #18534)

Dr. Wanda Weim, a licensed psychologist, received a request to testify in a custody proceeding on behalf of a man that she has been seeing in conjoint marital therapy until six months ago. The man wants Dr. Weim to provide the court with information about his parenting skills and his relationship with his wife and children. Dr. Weim contacts the man's wife, who refuses to give the psychologist permission to testify. Dr. Weim should:

A. Testify as requested by the husband

B. Testify only about matters pertaining to the husband

C. Testify only about matters for which she is qualified to make a judgment

D. Refused to testify

D. Refused to testify (Correct) In this situation, both clients (the husband and wife) method to give their permission before Dr. Weim can release confidential information. Therefore, he should refuse to testify (unless, of course, he is ordered to do so by the court). Multi-client situations create special considerations and concerns about confidentiality since it is not often possible to release information about one client without also revealing confidential information about another client.

(AATBS Ethics Domain Quiz #18455)

Dr.‭ ‬Sam Stone has been hired by a consulting firm to assist with an ongoing research project being conducted at a large company.‭ ‬Dr.‭ ‬Stone is to work directly under the project director,‭ ‬Dr.‭ ‬Thompson,‭ ‬a licensed organizational psychologist.‭ ‬Dr.‭ ‬Thompson asks Dr.‭ ‬Stone to review the preliminary report she has prepared for the project‭; ‬and,‭ ‬in doing so,‭ ‬Dr.‭ ‬Stone discovers that some of the reported data is misleading and may be inaccurate. ‭ ‬Dr.‭ ‬Stone expresses his concerns to Dr.‭ ‬Thompson and is told that‭ “‬it will be taken care of.‭” ‬Subsequently,‭ ‬Dr.‭ ‬Stone learns that the report was submitted to the company without any changes.‭ ‬Dr.‭ ‬Stone reviews the ethical guidelines and decides that pursuing the matter further isn't necessary. ‭ ‬Dr.‭ ‬Stone's conclusion:‬‬‬‬‬‬‬‬‬‬‬‬‬‬‬‬‬‬‬‬‬‬‬‬‬‬‬‬‬‬‬‬‬‬‬‬‬‬‬‬‬‬‬‬‬‬‬‬‬‬‬‬‬‬‬‬‬‬‬‬‬‬‬‬‬‬‬‬‬‬‬‬

a. is correct since this situation is not covered by ethical guidelines.

b. is correct since he has already attempted to resolve the matter in an informal manner as proscribed by ethical guidelines.

c. is correct since it is the project director’s responsibility to provide the company with complete and accurate data.

d. is incorrect because he has a responsibility to ensure that the company is provided with complete and accurate data.

d. is incorrect because he has a responsibility to ensure that the company is provided with complete and accurate data. (CORRECT) This answer is most consistent with the “spirit” of ethical guidelines and, more specifically with Standard 1.01 of the Ethics Code, which states that “if psychologists learn of misuse or misinterpretation of their work, they take reasonable steps to correct or minimize the misuse or misrepresentation”; and with Principle II.5 of the Canadian Code of Ethics, which states that psychologists “make every reasonable effort to ensure that psychological knowledge is not misused, intentionally or unintentionally, to harm others.” (Also see Standard 5.01 of the Ethics Code and Principle III.1 of the Canadian Code of Ethics.) This issue is addressed by Standard 1.01 of the APA’s Ethics Code and Principle II.5 of the Canadian Code of Ethics.

(AATBS Ethics Domain Quiz #18508)

A psychologist who already has an established therapeutic relationship with a child is asked to act as an expert witness in a divorce proceeding involving custody of the child.‭ ‬The psychologist's best course of action would be to:

A. decline the request to testify.

B. agree to testify only if she obtains consent from both parents.

C. agree to testify only if she can evaluate all of the involved parties.

D. agree to testify only if her familiarity with the child won''t bias her testimony.

A. decline the request to testify (CORRECT) Since the psychologist has a therapeutic relationship with the child, it would be in the child's best interests for the psychologist to refrain from testifying. Instead, the psychologist’s role should be to provide the child with continued support. B. agree to testify only if she obtains consent from both parents. Incorrect - Even if both parents consent to her testimony, the psychologist would be engaging in a multiple relationship which ordinarily should be avoided in custody cases. See Standard II.7 of the APA’s Guidelines for Child Custody Evaluations in Divorce Proceedings. C. agree to testify only if she can evaluate all of the involved parties. Incorrect - This would be the best course of action when a psychologist does not have a previous relationship with any of the involved parties. D. agree to testify only if her familiarity with the child won''t bias her testimony. Incorrect - One of the reasons that the psychologist should not testify is because it might be difficult to be objective.

(AATBS Ethics Domain Quiz #18549)

Ethical standards for varying professions differ with regard to their emphasis and perspective.‭ ‬As a psychologist working in a multidisciplinary setting,‭ ‬you should:‬‬‬‬‬‬

a. only work with fellow psychologists when developing professional and ethical standards for psychological services.

b. cooperate with other professionals when developing professional and ethical standards for psychological services.

c. consult with your local ethics committee for permission to work with other professionals when developing professional and ethical standards for psychological services. Incorrect Psychologists do not need to obtain permission to work with other professionals.

d. develop professional and ethical standards for mental health services that do not recognize professional differences.

b. cooperate with other professionals when developing professional and ethical standards for psychological services. (CORRECT) General Principle B of APA's Ethics Code states that “psychologists consult with, refer to, or cooperate with other professionals and institutions to the extent needed to serve the best interests of those with whom they work”; and the Values Statement of Principle IV of the Canadian Code of Ethics states that “in order to be responsible and accountable to society, and to contribute constructively to its ongoing development, psychologists need to be willing to work in partnership with others.” This answer is most consistent with these guidelines. Psychologists frequently consult with other professionals, such as medical doctors, lawyers, and teachers. It is imperative that they be aware that every profession has professional and ethical standards that must be maintained.

a. only work with fellow psychologists when developing professional and ethical standards for psychological services - Incorrect It is unrealistic to presume that it is possible to limit decision-making to psychologists only. There are many situations, especially in a multidisciplinary setting, where consultation with other professionals is necessary.

c. consult with your local ethics committee for permission to work with other professionals when developing professional and ethical standards for psychological services. - Incorrect Psychologists do not need to obtain permission to work with other professionals.

d. develop professional and ethical standards for mental health services that do not recognize professional differences. Incorrect Each profession has different professional and ethical responsibilities, and it is not advisable (and often not possible) to homogenize the responsibilities of different professions.

(AATBS Ethics Domain Quiz #18553)

Dr. Vic Vasquez, director of a private counseling center, prepares and distributes a brochure describing the centers services. The brochure is available to potential clients and other interested people, and it's text includes a list of the centers staff members and their credentials. Specifically, it indicates that five of the six members have doctoral degrees from accredited universities and that the member who does not have a degree in a "Ph.D. candidate." The designation of the non-degreed staff member is:

A. Acceptable because it accurately reflects the staff member's status

B. Acceptable because it was not meant to be deceptive

C. Acceptable because the issue is not addressed by ethical guidelines

D. Unacceptable because it is potentially misleading

D. Unacceptable because it is potentially misleading (Correct) Ethical guidelines caution against presenting one's credentials in a way that could be misleading. Dr. Vasquez should not have listed the staff member as a "Ph.D. candidate" because it is possible that many people will not understand what this means. The situation described in this question is addressed by Standard 5.01 of the APA's Ethics Code and Principle III.2 of the Canadian Code of Ethics.

(AATBS Ethics Domain Quiz #18567)

A psychologist hired by the court to evaluate the two young children of couple who are seeking a divorce. The psychologist job will be to evaluate both children and report its findings to the court with any recommendations regarding custody he can make on the basis of his findings. Through her attorney, the mother learns that the children have both revealed information to the psychologist that presents her in an unfavorable light. The mother contact the psychologist and says that since her children are both minors, she is the holder of the privilege and is, therefore, instructing the psychologist not to report the unfavorable information to the court. The psychologist should:

A. Comply with the mother's request to not reveal the unfavorable information since she is the holder of the privilege

B. Provide the court with the information about the results of the evaluation as long as he believes is evaluation is non-biased

C. Provide the court with the information he feels is in the best interest of the children

D. Refused to testify unless court order to do so

C. Provide the court with the information he feels is in the best interest of the children (Correct) As noted by E.J. Kermani (Handbook of Psychiatry and the law, Chicago, Year Book Medical Publ., Inc., 1989), the concept of privilege "specifies that the parents" right must yield to the best interest of the child, regardless of who requested the evaluation in a custody case. In other words, parents cannot claim privilege or refrain from waiving privilege in legal matters pertaining to custody. In custody cases, a psychologist must always put the best interest of the child first.

D. Refused to testify unless court order to do so (Incorrect) the psychologist was hired by the court, so he does not need a court order in order to testify.

(AATBS Ethics Domain Quiz #18552)

Which of the following best describes the status of professional (therapy) records?

A. The physical record belongs to the client

B. The physical record belongs to the therapist

C. The physical record belongs to the therapist, but the information contained therein belongs to the client.

D. The physical record and the information contained therein belong jointly to the therapist and the client.

C. The physical record belongs to the therapist, but the information contained therein belongs to the client. (Correct) While the actual physical record is the property of the therapist, laws require therapists to make the information contained in them available to the client in most situations. This does not mean that a therapist must release the physical record to a client but, instead, that the therapist mat be required to show the client the record or provide the client with a summary or copy of it. The laws regarding professional records are complex and vary somewhat from jurisdiction to jurisdiction. Keep in mind that, when answering questions like this one, you’ll want to pick a general answer that is likely to apply to most (or all) jurisdictions.

(AATBS Ethics Domain Quiz #18439)

Dr.‭ ‬La-Keysha Leonard,‭ ‬a licensed psychologist,‭ ‬has been seeing a prison inmate in group therapy for several months and is asked by the parole board to evaluate the inmate to assist with their decision regarding his parole.‭ ‬As an ethical psychologist,‭ ‬Dr.‭ ‬Leonard should:

A. agree to evaluate the inmate.

B. agree to evaluate the inmate only if she believes she can do so objectively and without bias.

C. agree to do so only if she is allowed to explain the purpose of the evaluation and the limits on confidentiality to the inmate.

D. refuse to evaluate the inmate for the purpose of parole.

D. refuse to evaluate the inmate for the purpose of parole - (CORRECT) This answer is most consistent with ethical guidelines prohibiting psychologists from becoming involved in multiple relationships that might impair their objectivity and with the APA’s Specialty Guidelines for Forensic Psychologists, which warns psychologists about the need for caution in dual relationships and roles in forensic settings (the actions described in answer a, b, and c do not constitute sufficient caution). The situation described in this question is addressed by general ethical guidelines for multiple relationships and in the APA’s Specialty Guidelines for Forensic Psychologists.

(AATBS Ethics Domain Quiz #18557)

When conducting research with human participants,‭ ‬a psychologist should be aware that:‬‬‬‬

a. participants must be debriefed as soon as possible after their participation in the study,‭ ‬especially when the study has involved deception. ‬‬‬‬

b. participants must be debriefed after their participation in the study only if the psychologist believes that not doing so will result in harm to them.

c. participants must be debriefed after their participation only if the study places them at‭ “‬high risk.‭” ‬‬‬‬‬‬‬‬

d. debriefing of research participants is at the discretion of the psychologist.

a. participants must be debriefed as soon as possible after their participation in the study,‭ ‬especially when the study has involved deception. ‬‬‬‬(CORRECT) Standard 8.08 of the Ethics Code requires psychologists to inform participants promptly following their participation in the study about its nature, results, and conclusions; and Principle III.26 of the Canadian Code of Ethics requires psychologists to debrief participants as soon as possible after their involvement in a study when there has been incomplete disclosure about the study’s purpose. Consequently, of the answers given, this one is most consistent with ethical guidelines. Debriefing of research participants is addressed in Standard 8.08 of the APA’s Ethic Code and Principles II.44 and III.26 of the Canadian Code of Ethics.

b. participants must be debriefed after their participation in the study only if the psychologist believes that not doing so will result in harm to them.

c. participants must be debriefed after their participation only if the study places them at‭ “‬high risk.‭” ‬‬‬‬‬‬‬‬

d. debriefing of research participants is at the discretion of the psychologist.

(AATBS Ethics Domain Quiz #18486)

As defined in the APA's‭ ‬General Guidelines for Providers of Psychological Services and the CPA’s‭ ‬Practice Guidelines for Providers of Psychological Services,‭ “‬psychological services‭” ‬include all of the following‭ ‬except‭?‬‬‬‬‬‬‬‬‬‬‬‬‬‬‬‬‬‬

a. the conduct of scientific research

b. program development

c. consultation related to assessment and intervention

d. supervision of psychological services

a. the conduct of scientific research (Correct Answer) The conduct of scientific research is not included in the definition of psychological services. The APA and CPA guidelines are both very specific about the activities that constitute “psychological services.”

b. program development (Incorrect) Incorrect Program development is included.

c. consultation related to assessment and intervention - Incorrect Consultation is included.

d. supervision of psychological services - Incorrect Supervision is included. (AATBS Ethics Domain Quiz #18532)

You are subpoenaed to appear at a deposition to testify about a former client of yours.‭ ‬During the deposition,‭ ‬the attorney for the opposing party asks you to provide confidential client information.‭ ‬You should:‬‬‬‬‬‬

a. refuse to provide the information without a court order.

b. assert the privilege unless the client has signed a waiver.

c. provide the information if you believe it is pertinent to the case.

d. provide the information since privilege is waived in this situation.

b. assert the privilege unless the client has signed a waiver. (Correct Answer)‬ Confidential client information should be revealed in a legal proceeding only when the client has signed a waiver or when the court has issued an order indicating that privilege is waived. If the client’s attorney is present, you may ask him or her if you should answer the question. However, if the attorney is not present, you should not provide the information without a waiver from the client but, instead, should assert the privilege on behalf of the client.

a. refuse to provide the information without a court order. - Incorrect You may provide the information if the client has signed a waiver.

(AATBS Ethics Domain Quiz #18538)

You have been hired to assist with a research project on process variables in group psychotherapy.‭ ‬The psychologist who is the principal investigator tells you to discourage participants from leaving the group prior to the end of the study because,‭ ‬if too many participants drop out,‭ ‬the study's potential usefulness will be seriously compromised.‭ ‬As an ethical psychologist,‭ ‬you should:

A. refuse to assist with the research project.

B. immediately file a complaint against the psychologist with the psychology licensing board or ethics committee.

C. write a letter to the psychologist,‭ ‬indicating the relevant ethical standards and offering to discuss the matter with him.

D. follow the psychologist''s instructions since,‭ ‬by‭ "‬discouraging" ‬participants from leaving the group,‭ ‬you are not actually coercing them to participate.

C. write a letter to the psychologist,‭ ‬indicating the relevant ethical standards and offering to discuss the matter with him - (CORRECT) By offering to discuss the matter with the psychologist, you are attempting to resolve the matter informally, which is consistent with the requirements of the ethical guidelines. Standard 8.02 of the APA’s Ethics Code and Principle I.30 of the Canadian Code of Ethics apply to this situation. Both require psychologists to allow participants to withdraw from a research study at any time.

A. refuse to assist with the research project - (Incorrect) Resigning from the project would not correct the ethical violation.

B. immediately file a complaint against the psychologist with the psychology licensing board or ethics committee - (Incorrect) Although a report may eventually be necessary, it is not the best first step since this matter may be amenable to an informal resolution (answer c).

D. follow the psychologist''s instructions since,‭ ‬by‭ "‬discouraging" ‬participants from leaving the group,‭ ‬you are not actually coercing them to participate - (Incorrect) This would be unethical, and in so doing, you would be violating the provisions of Standards 1.04 and 1.05 of the Ethics Code and Principles II.40 and II.41 of the Canadian Code of Ethics, which require psychologists to take action when a colleague acts in an unethical way.

(AATBS Ethics Domain Quiz #18443)

A colleague,‭ ‬a licensed psychologist,‭ ‬confides in you that he has just ended a sexual relationship with one of his clients.‭ ‬As he describes the client,‭ ‬you realize that she is someone you referred to him about‭ ‬18‭ ‬months ago.‭ ‬As an ethical psychologist,‭ ‬you should:‬‬‬‬‬‬‬‬‬‬‬‬‬‬‬‬‬‬‬‬‬‬‬‬

a. file a complaint against the colleague immediately with the Ethics Committee.

b. file a complaint against the colleague with the Ethics Committee after informing him that you intend to do so.

c. discuss the matter further with the colleague to see what course of action he plans to take.

d. contact the client and discuss the matter with her.

c. discuss the matter further with the colleague to see what course of action he plans to take. (CORRECT) Ethical guidelines require psychologists to address ethical violations by other psychologists but to do so in a way that does not violate the confidentiality of the involved client. Therefore, this is the best course of action of those presented in the answers to this question. Ethical violations by colleagues are addressed by Standards 1.04 and 1.05 of the APA’s Ethics Code and Principles II.40 and 41 of the Canadian Code of Ethics. a. file a complaint against the colleague immediately with the Ethics Committee. - Incorrect Filing a complaint with the Ethics Committee would be unethical if it involved a breach of the client's confidentiality.

b. file a complaint against the colleague with the Ethics Committee after informing him that you intend to do so. - Incorrect See explanation for responses a and c.

d. contact the client and discuss the matter with her. - Incorrect The question does not say that you have a current professional relationship with the client and, even if you did, it probably would not be appropriate to confront her with this from both an ethical and clinical perspective.

(AATBS Ethics Domain Quiz #18550)

After three sessions with a therapy client,‭ ‬Dr.‭ ‬Leonard Lykowski realizes that he’s feeling somewhat hostile toward the client because she reminds him of his wife who he’s currently divorcing.‭ ‬Dr.‭ ‬Lykowski’s best course of action would be to:‬‬‬‬‬‬‬‬‬‬‬‬

a. refer the client to another therapist without telling the client the reason for the referral.

b. refer the client to another therapist after discussing the reason with her.

c. recognize that his feelings are the result of countertransference and continue to work with the client.

d. consult with another psychologist to determine whether or not to continue seeing the client in therapy.

d. consult with another psychologist to determine whether or not to continue seeing the client in therapy. (CORRECT) Consultation with another professional is ordinarily the best course of action whenever a therapist is concerned that a personal problem might interfere with his/her ability to provide a client with adequate therapy. In the situation described in this question, Dr. Lykowski’s personal problems may be affecting his objectivity.

c. recognize that his feelings are the result of countertransference and continue to work with the client. - Incorrect Although Dr. Lykowski’s feelings do represent countertransference, he would want to seek consultation to determine if those feelings are negatively impacting (or are likely to negatively impact) his ability to provide the client with appropriate services.

(AATBS Ethics Domain Quiz #18457)

Following the retirement of his business partner,‭ ‬Dr.‭ ‬Sheldon Smythe is the only psychologist in a small town,‭ ‬and the next closest mental health professional works in a town that is over‭ ‬100‭ ‬miles away.‭ ‬Several of Dr.‭ ‬Smythe’s new clients have problems for which he has limited training and experience.‭ ‬Dr.‭ ‬Smythe should:‬‬‬‬‬‬‬‬‬‬‬‬‬‬‬‬‬‬‬‬‬‬‬‬‬‬‬

a. refuse to see the clients.

b. see these clients but use only interventions that he has experience using.

c. see the clients and obtain supervision or consultation by telephone.

d. agree to see these clients only if they are experiencing a crisis.

c. see the clients and obtain supervision or consultation by telephone. (CORRECT) It is acceptable and desirable for psychologists to acquire new skills and to treat new problems – but only when appropriate supervision or consultation is obtained. The situation described in this question is a complex one, but this response is the best one given the circumstances (the next closest mental professional works in a town that is over 100 miles away) and the alternatives provided in the other answers. Although psychologists should ordinarily provide services that are within the boundaries of their training and experience, there are exceptions.

(AATBS Ethics Domain Quiz #18452)

A psychologist in private practice is going through a difficult divorce and,‭ ‬as a result,‭ ‬has started drinking and is often depressed. He should:‬‬‬‬‬‬‬‬‬

a. plunge deeper into his work to take his mind off his own problems.

b. seek professional consultation and consider temporarily suspending his practice.

c. explain the situation to his clients and offer to refer them to other therapists.

d. immediately refer his clients to other therapists.

b. seek professional consultation and consider temporarily suspending his practice - CORRECT Psychologists do not necessarily have to abandon their professional practice when experiencing a period of personal distress. However, when personal problems may be adversely affecting the welfare of their clients, psychologists must take appropriate action. This often includes “obtaining professional consultation or assistance ... to determine whether they should limit, suspend, or terminate their work-related duties” [Ethics Code, Standard 2.06(b)]. This issue is addressed by Standard 2.06(b) of the APA’s Ethics Code and Principle II.11 of the Canadian Code of Ethics. d. immediately refer his clients to other therapists. - Incorrect This may be necessary, but the best initial step would be to seek consultation to determine the most appropriate course of action.

(AATBS Ethics Domain Quiz #18489)

Dr. Nancy Noonan plans to conduct a study that will involve testing and interviewing psychiatric inpatients. Before beginning the study, she must obtain:

A. Informed consent from the hospital administrator

B. Informed consent from the patient's legal guardian

C. Informed consent from the patient's legal guardian and assent from the patients

D. Assent from the patient's

C. Informed consent from the patient's legal guardian and assent from the patients (Correct) The answers given to this question imply that the patients are unable to give informed consent to their participation in the study since this option is not provided in any of the answers. Therefore, this is the best answer of those given. In most cases, whenever a person is unable to provide an informed consent for participation in research (i.e., because he/she is incapable of doing so), a psychologist must obtain consent from the person legal guardian and assent (agreement) from the person. Standards 3.10 and 8.02 of the APA's Ethics Code and Principle I.35 of the Canadian Code of Ethics apply for this situation described in this question.

(AATBS Ethics Domain Quiz #18516)

Which of the following techniques would be most useful for evaluating the impact of an intervention on a patient's quality of life?

A. Cost – feasibility analysis

B. Cost – minimization analysis

C. Cost – utility analysis

D. Cost – benefit analysis

C. Cost - utility analysis (Correct) A cost – utility analysis is conducted to compare the costs and patient outcomes of an intervention, with outcomes including patient's duration and quality of life. Cost analysis is conducted to determine the optimal allocation of financial resources. Experts distinguish between the four methods of cost analysis listed in the answers to this question. Additional information on these methods is provided in the ethics and professional issues chapter of the written study materials.

A. Cost - feasibility analysis (Incorrect) cost – feasibility analysis is used to determine the feasibility of an intervention be on the monetary and other resources that requires.

B. Cost - minimization analysis (Incorrect) as its name implies, cost – minimization analysis is conducted to identify the least costly option among several options that produces equivalent outcomes.

D. Cost - benefit analysis (Incorrect) as its name implies, a cost – benefit analysis is used to determine the relative costs and benefits of a particular intervention.

(AATBS Ethics Domain Quiz #18564)

The Examination for Professional Practice in Psychology (EPPP) is best described as a:

A. Predictor of performance as a professional psychologist

B. Measure of basic knowledge of psychology

C. Measure of basic competence

D. Method for protecting the profession of psychology

B. Measure of basic knowledge of psychology (Correct) The Association of State and Provincial Psychology Boards (ASPPB) describes the EPPP as a measure of "the knowledge that should have been acquired by any candidate who is seeking licensure to practice psychology." The EPPP is a requirement for licensure in the United States and Canada.

(AATBS Ethics Domain Quiz #18542)

Dr. Brian Bertram, a licensed psychologist, is hired by the owner of the company to evaluate employees who are not doing well on the job to determine if they would benefit from counseling or additional training. In this situation, Dr. Bertram should:

A. Reveal the information obtained from each employee to the owner of the company since she is the client

B. Reveal information about each employee only after obtaining signed consent from employees since they are the client

C. Not discussed confidentiality with the employees since to do so might limit their willingness to reveal information to him

D. Inform each employee prior to the evaluation of the potential limits on confidentiality

D. Inform each employee prior to the evaluation of the potential limits on confidentiality (Correct) Standard 3.07 of the APA's Ethics Code states that "when psychologist agreed to provide services to a person or entity at the request of a third party, psychologist attempted clarify at the outset of the services the nature of the relationship with all individuals or organizations involved. This clarification includes… The fact that there may be limits to confidentiality." Principle I.26 of the Canadian Code of Ethics contains similar language. In the situation described in this question, the company does have a right to relevant information, but this does not free the psychologist from the obligation to inform employees about the limits of confidentiality and the company about what kind of employee information will be revealed.

(AATBS Ethics Domain Quiz #18559)

A 26-year-old client you have been seeing for two months unexpectedly admits to use that he had sexual relations with the 15-year-old daughter of his neighbor six months ago. He said that he knows what he did was wrong and he'll never do it again. You should:

A. Maintain the client confidentiality

B. Report the client to the appropriate authorities

C. Alert the parents of his victim

D. Report to client to the appropriate authorities alert the parents of his victim

B. Report the client to the appropriate authorities (Correct) You would want to contact the appropriate authorities in the situation, but contacting the parent of the victim (answers C and D) would represent a breach of the client confidentiality. In cases of known and suspected child abuse, a psychologist legally and ethically required to make a report to the appropriate authorities.

(AATBS Ethics Domain Quiz #18507)

A psychology professor at a large university asked a graduate student to assist on a research project in a field of interest to the student. The student will be required to spend about 25 hours a week on the project and will be paid on an hourly basis. With regard to publication credit, the professor should do which of the following when preparing a journal article about the study?

A. You should include the student of the co-author of the student is working more than half time on the project

B. You should note the students contributions in a footnote for introductory statement since the student is essentially an employee

C. He does not need to mention the students contribution since the student is an employee

D. He should acknowledge the student in a manner consistent with her contribution to the project

D. He should acknowledge the student in a manner consistent with her contribution to the project (Correct) This response is most consistent with ethical guidelines. Standard 8.12 (b), for example, states that "principle authorship and other publication credits accurately reflect the relative scientific or professional contributions of the involved, regardless of their relative status." Standard 8.12 of the APA's Ethics Code and Principles III.7 of the Canadian Code of Ethics apply to the situation described in this question.

A. You should include the student of the co-author of the student is working more than half time on the project (Incorrect) this would be necessary only if the student made a major contribution to the project.

B. You should note the students contributions in a footnote for introductory statement since the student is essentially an employee in (Incorrect) this would be sufficient only if the student made a minor contribution to the project. The fact that the student is being paid does not preclude publication credit.

C. He does not need to mention the students contribution since the student is an employee (Incorrect) the fact that the student is being paid for her participation in the research project is not precluded publication credit.

(AATBS Ethics Domain Quiz #18551)

A psychologist is hired by a court to evaluate the defendant during the course of a criminal proceeding. The psychologist can subsequently include information about the defendant in a book that she is writing on "the criminal mind":

A. As long as the defendant was warrened of the limits of confidentiality at the time of the evaluation.

B. As long as the information included in the book is included in the public record of the case.

C. As long as the psychologist determines that the value of the information warrants its inclusion in the book

D. Only at the psychologist has received the permission of the defendants for the defendant's legal representatives to do so.

D. Only at the psychologist has received the permission of the defendants for the defendant's legal representatives to do so (Correct) this is the only response that mentions obtaining consent from the defendant, which would be required for the psychologist who conducted the evaluation of the defendant. It is unethical to use evaluation results in a manner that is inconsistent with the original purpose of the evaluation unless the client has given consent. See, e.g., the APA's Specialty Guidelines for Forensic Psychologists.

A. As long as the defendant was warrened of the limits of confidentiality at the time of the evaluation (Incorrect) this would be insufficient

B. As long as the information included in the book is included in the public record of the case (Incorrect) this is true for individuals not involved in the evaluation of the defendant but not for the psychologist who conducted the evaluation

C. As long as the psychologist determines that the value of the information warrants its inclusion in the book (Incorrect) this is untrue

(AATBS Ethics Domain Quiz #18484)

To be consistent with ethical requirements,‭ ‬David DeFoe,‭ ‬a psychology intern,‭ ‬must do which of the following when a supervisor has legal responsibility for his work‭?

A. He must tell his clients that he is a psychology intern.

B. He must tell his clients that his work is being supervised.

C. He must provide clients with information about his education and experience.

D.He must indicate his status on his business card.

A. He must tell his clients that he is a psychology intern - (CORRECT) This answer is most consistent with ethical guidelines which require interns (trainees) to inform their clients of their professional status. Note that Standard 10.01(c) of the Ethics Code also requires that Mr. DeFoe provide clients with the name of his supervisor.

B. He must tell his clients that his work is being supervised - Incorrect Telling clients that he is being supervised would not be adequate since this would not clearly indicate that he is an intern.

(AATBS Ethics Domain Quiz #18514)

The term “insanity” is a:

a. popular (lay) term for a mental disorder.

b. legal, non-psychiatric term for certain types of mental disorders.

c. psychiatric term that refer to a psychotic disorder with an unknown etiology

d. psychiatric term that refers to an organic mental disorder.

b. legal, non-psychiatric term for certain types of mental disorders. (Correct Answer) The term "insanity" is used in the legal arenas and is not a psychiatric term. Generally, a defendant who pleads not guilty by reason of insanity is contending that he was precluded by a mental disease or defect from understanding the differences between right and wrong. Insanity is a legal term that varies somewhat in terms of its definition from jurisdiction t jurisdiction.

(AATBS Ethics Domain Quiz #18449)

After a year in private practice, you hire a new licensed psychologist whom you will provide with an office and secretarial services. You will also refer clients to the psychologist and pay him 50% of the fees collects from those clients. This arrangement is:

A. Unethical because it involves barter

B. Unethical because it involves being paid for client referrals

C. Ethical as long as the psychologist is being compensated in an equitable way

D. Ethical because, as an employer, you can determine the conditions of employment

C. Ethical as long as the psychologist is being compensated in an equitable way (Correct) of the responses given, this one is the best. Referral fees are not entirely prohibited by ethical guidelines, which will out responses A and B; and response D is not an accurate description of an employer's ethical responsibilities. This issue is addressed in Standard 6.07 of the Ethics Code and is discussed in the Ethics Manual of the written study materials. The arrangement described in this question seems somewhat dubious in terms of acceptability, but the facts presented do not clearly indicate an ethical violation.

(AATBS Ethics Domain Quiz #18433)

A woman in comes to family therapy with her three daughters, ages 10, 13, and 19, all of whom live at home with the woman and their father. During the third therapy session, the 19-year-old tells you that her father began sexually molesting her when she was 13. When you ask the younger girls if their father has ever abuse them, they are reluctant to say anything about their father, and the mother adamantly denies any knowledge of abuse. In this situation, you should:

A. Advise the mother to contact the appropriate authorities

B. File a report with the appropriate authorities yourself

C. Meet with the father to determine if he is abusing the younger daughters

D. Maintain confidentiality since the known victim of abuse is over 18

B. File a report with the appropriate authorities yourself (correct) although psychologists are not usually required to make a report when the individual is over the age of 18, in that situation described in that question, you would have reasonable suspicion that the younger sisters are also victims of abuse. Therefore, you would be legally required to file a report with the appropriate authorities. Additional information about reporting child abuse is provided in the ethics manual of the written study materials. Whenever you acquire a reasonable suspicion in your professional capacity that a minor is being abused, you have a legal obligation to report the abuse.

(AATBS Ethics Domain Quiz #18463)

Faaris F. terminated therapy with Dr. Pamela Prince several months ago but calls to request an appointment because he is now very lonely and oppressed and feels "like he wants to die."In the past two months, Dr. Prince has been reducing her caseload because she is planning to retire. The best course of action in this situation is for Dr. Prince to:

A. Explain to Faaris that she is reducing her caseload but that she is willing to counsel him by phone in case of emergency

B. See Faaris in therapy until the crisis has passed and then provide him with appropriate referrals

C. Explain to Faaris that she is no longer accepting clients and referred him to a college

D. Provide Faaris with several referrals and ask him to call her if none of those turn out to be satisfactory

B. See Faaris in therapy until the crisis has passed and then provide him with appropriate referrals (Correct) since the client seems to be experiencing a crisis (he feels "like he wants to die"), this course of action takes his welfare into account and is most consistent with the "spirit"of the ethical guidelines. See, for example, standard 3.04 and 10.09 of the APA's Ethics Code and Principles II.2 and II.34 of the Canadian Code of Ethics. The situation described in this question is not explicitly addressed by ethical guidelines, so the best strategy is to choose the answer that best addresses the clients welfare.

(AATBS Ethics Domain Quiz #18454)

Dr. Bettina Browne, a licensed psychologist, undertakes a child custody evaluation at the request of the child's mother. The father agrees to be evaluated also, & Dr. Browne obtains signed informed consent from both parents before beginning her assessment. Dr. Browne adminsters the MMPI & Rorschach tests to both parents. The mother obtains significantly elevated scores on the MMPI's K & L scales, suggesting that she tends to lie and be excessively defensive, and these tendencies are confirmed by her Rorschach responses. No significant abnormality is suggested by the father's MMPI and Rorschach results. On the babsis of these test results, Dr. Browne recommends that the mother not be awarded custody of the child. Dr. Browne:

A. has acted ethically because her evaluation included both parents.

B. has acted ethically because the parents signed informaed consents before the evaluation.

C. has acted unethically because of her use of the MMPI and Rorschach tests as methods of evaluating the mother's parental suitability.

D. has acted unethically because she made specific recommendations regarding custody rather than simply reporting the results of the evaluation.

A. has acted ethically because her evaluation included both parents.- Incorrect although the psychologist acted appropriately by evaluating both parents prior to making a recommendation, response c is a better answer.

B. has acted ethically because the parents signed informaed consents before the evaluation. - Incorrect obtaining signed informed consent does not overcome the problem of using the MMPI and Rorschach as predictors of parental functioning, a purpose for which they have not been validated. 

C. has acted unethically because of her use of the MMPI and Rorschach tests as methods of evaluating the mother's parental suitability. - CORRECT A psychologist is ethically required to use tests only for the purposes for which they have been validated. Based on the information given, it appears that Dr. Browne has acted unethically by basing her custody recommendation on MMPI and Rorschach test results since these tests have not been validated for this purpose, Dr. Browne's recommendation should have been nased on more than the results of only two tests.

D. has acted unethically because she made specific recommendations regarding custody rather than simply reporting the results of the evaluation. - Incorrect Recommendations are acceptable as long as they are based on a thorough evaluation of all parties. 

(AATBS Ethics Domain Quiz #18440)

A psychology intern offers his supervisor a chance to invest in a real estate syndicate that is being arranged by the interns father as a tax shelter. The supervisor:

A. Would be foolish to refuse

B. Should invest but only in his wife's name

C. Should refuse to invest

D. Should report the intern to the ethics committee

C. Should refuse to invest (Correct) multiple relationships are addressed in standard 3.05 of the APA's Ethics Code and Principle III.3 of the Canadian code of ethics. Standard 3.05 states that "a psychologist refrains from entering into a multiple relationship if the multiple relationship reasonably be expected to bear the psychologist objectivity, competence, or effectiveness in performing his or her functions as a psychologist, or otherwise risk exploitation or harm to the person with whom the professional relationship exists." Principle III.3 of the Canadian code of ethics includes a similar requirement. The situation described in this question represents a multiple (dual) relationship that the potential he impact the psychologists ability to remain objective when acting as a supervisor for the intern.
D. Should report the intern to the ethics committee (Incorrect) it would be a better strategy for the supervisor to educate the intern about multiple relationships then to report the intern to the ethics committee.

(AATBS Ethics Domain Quiz #18490)

During the course of treatment, a woman reports to her therapist, Dr. Emil Emerson, that her husband hit somebody, causing the victim a mild concussion. If Dr. Emerson maintains the client confidentiality, he has acted: A. Ethically since confidentiality in this situation should not be breached B. Ethically since the women's report is only hearsay C. Unethically since he should have reported the incident to the appropriate authorities

D. Unethically since he should have contacted the intended victim

A. Ethically since confidentiality in this situation should not be breached (Correct) a breach of client confidentiality is acceptable, in some cases, mandated by law. However, this situation involves a past incident involving the client's husband, and a breach of confidentiality is not acceptable or required. See, for example, standard 4.05(b) of the APA's Ethics Code, which states that exceptions to client confidentiality are permitted only when it reaches necessary to avoid violating the law, or when breaching confidentiality would "protect the client/patient, psychologist, or others from harm." The situation described in this question is not one that mandates a breach of confidentiality.

(AATBS Ethics Domain Quiz #18496)

Dr. Beatrice Bowen, a licensed psychologist, has been contacted by a company to evaluate job applicants. The job will involve administering projective tests to determine if applicants for an accountant  job have a "homosexual orientation." Dr. Bowen should: A. Not accept the job because the evaluation she is being requested to conduct represents an invasion of privacy B. Not accept this job because projective tests have been shown to be unreliable and invalid C. Accept the job and administered test as required but explain the purpose to all applicants before doing so

D. Accept the job, administer the test as requested, and report the results to the company

A. Not accept the job because the evaluation she is being requested to conduct represents an invasion of privacy (Correct) this answer is the most consistent with ethical guidelines. Standard 9.04(b) of the APA's Ethics Code, for example, states that "psychologists may refrain from releasing test data to protect a client/patient or others from substantial harm or misuse or misrepresentation of the data or the test"; and principal I.38 of the Canadian code of ethics states that psychologists "take care not to infringe… On the personally, developmentally, or culturally defined private space of individuals or groups, unless clear permission is granted to do so." A "homosexual orientation"is not relevant to the ability to perform an accountant job successfully and to seek sex information as part of a job-related assessment is clearly an invasion of privacy.

(AATBS Ethics Domain Quiz #18497)

Dr. Delores Del Rio in seeing Jeff J. in therapy for four months. For the last month, Jeff has complained about the progress he is making in therapy and, Dr. Del Rio has responded by working with him to modify treatment goals. In the last therapy session, however, Jeff again stated that he is not happy with therapy. Dr. Del Ro's best course of action would be to:  A. Recognized that Jeff's concerns are a normal reaction to the changes he is making as the result of therapy B. Make a clinical decision as to whether or not Jeff is actually benefiting from therapy and, if he is, encourage him to continue with therapy for at least several more sessions C. Discuss Jeff's continued dissatisfaction with him and the possibility of terminating therapy

D. Tell Jeff that she is ethically required to terminate therapy with him since he is not satisfied with the progress he is making

C. Discuss Jeff's continued dissatisfaction with him and the possibility of terminating therapy (Correct) this is the best answer of those given because it takes the clients welfare into consideration and is most consistent with ethical guidelines regarding termination of professional services [See standard 10.10(a) of the APA's Ethics Code or PrinciplesII.37 of the Canadian Code of Ethics]. This question addresses the issue of terminating therapy when a client does not seem to be benefiting from it.(AATBS Ethics Domain Quiz #18501)

Dr.‭ ‬Donald Dreadmire is starting a therapy group for recently divorced men.‭ ‬He tells the editor of the local newspaper‭ (‬who has just divorced his wife‭) ‬that he can attend the group for free if the editor includes an article about the program in the newspaper.‭ ‬Dr.‭ ‬Dreadmire has acted:‬‬‬‬‬‬‬‬‬‬‬‬‬‬‬‬‬‬ a. ethically since his request does not violate ethical guidelines.  b. ethically as long as the editor accurately describes the program.  c. unethically because his request is exploitative and violates ethical guidelines. 

d. unethically only if the arrangement negatively impacts his objectivity when working with the editor in therapy. 

c. unethically because his request is exploitative and violates ethical guidelines. (Correct Answer) Of the answers given, this one is most consistent with ethical guidelines.  Standard 5.02(b) of the Ethics Code prohibits psychologists from compensating “employees of press, radio, television, or other communication media in return for publicity in a news item”; and Principle III.31 of the Canadian Code of Ethics prohibits psychologists from exploiting “any relationship established as a psychologist to further personal, political, or business interests at the expense of the best interests of their clients.” This issue is directly addressed by Standard 5.02(b) of the APA’s Ethics Code and indirectly addressed by Principle III.31 of the Canadian Code of Ethics.

(AATBS Ethics Domain Quiz #18503)

You are hired as an adjunct professor to teach a course at a graduate school of professional psychology. In terms of ethical requirements, you are required to ensure that: A, Accurate information about the course content is avalible for all interested paties. B. The course content sddresses the most recent research and theory in the area. C. THe course content addresses the interests of the students who enroll in the class.

D. Students are given and opportunity to evaluate you at the end of the term. 

A, Accurate information about the course content is avalible for all interested paties (Correct) Ethical guidelines explicitly require psychologists to provide interested parties(in case, the school administration and the students) with accurate and complete information about course content. This issues is addressed by Standard 7.02 of the APA's Ethics Code and PRinciles III.5 of the Canadian Code of EThics.  B. The course content sddresses the most recent research and theory in the area (Incorrect) Although presenting the most recent research and theory is usually a good idea, the importance of doing so would depend on the nature of the class. For example, recent research and theory woud not be covered in a "history of psychology" class. C. THe course content addresses the interests of the students who enroll in the class (Incorrect) The interests of the students do not necessarily coincide with the content that, you as the instructor, believe is important to present.

D. Students are given and opportunity to evaluate you at the end of the term (Incorrect) Providing students with opportunities to evaluate the course is desirable but is not required by ethical guidelines.

(AATBS Ethics Domain Quiz #18504)

Which of the following is most likely a violation of ethical guidelines?‬‬‬ a. a brochure containing client testimonials mailed to potential attendees at a conference for adult children of alcoholics b. an advertisement offering a free initial therapy session that is published in the campus newspaper just prior to final exams  c. uninvited in-person solicitations for therapy of hurricane victims that destroyed their homes

d. a display ad for psychotherapy services in the yellow pages of the local telephone directory

c. uninvited in-person solicitations for therapy of hurricane victims that destroyed their homes (CORRECT) In-person solicitation of individuals who are vulnerable to undue influence is explicitly prohibited by Standard 5.06 of the APA’s Ethics Code and is inconsistent with the “spirit” of the Canadian Code of Ethics (see, e.g., Principles I.27 and III.31). Of the advertisements/solicitations described in the responses, only one is clearly prohibited by ethical guidelines. a. a brochure containing client testimonials mailed to potential attendees at a conference for adult children of alcoholics - Incorrect Testimonials are acceptable as long as they are not solicited from clients who are susceptible to undue influence (see Standard 5.05 of the APA’s Ethics Code).  b. an advertisement offering a free initial therapy session that is published in the campus newspaper just prior to final exams - Incorrect Free initial sessions are not prohibited by ethical guidelines.

d. a display ad for psychotherapy services in the yellow pages of the local telephone directory - Incorrect Ads in the yellow pages of the phone directory are not prohibited by ethical guidelines.

(AATBS Ethics Domain Quiz #18528)

Dr.‭ ‬Harry Holliday,‭ ‬a clinical psychologist,‭ ‬is planning to take a three-week cruise.‭ ‬In terms of his clinical practice,‭ ‬Dr.‭ ‬Holliday’s‭ ‬best course of action would be to:‬‬‬‬‬‬‬‬‬‬‬‬‬‬ a. discuss his vacation plans with his current clients ahead of time so that they know he’ll be unavailable during that time.  b. give his clients a phone number where he can be reached.  c. give his clients the phone number of a colleague who has agreed to accept phone calls in emergencies. 

d. hire a colleague to take over his appointments while he is on vacation.

c. give his clients the phone number of a colleague who has agreed to accept phone calls in emergencies. (Correct Answer)‬ Of the answers given, this one makes the most sense in terms of the clients' welfare and Dr. Holliday's interests. Ethically, Dr. Holliday would not be required to be available by phone to his clients during his vacation or to provide alternative ongoing services but would be required to provide the clients with a contact for emergency situations. Dr. Holliday should, of course, make some kind of arrangements for his clients while he is on vacation.

(AATBS Ethics Domain Quiz #18529)

The researchers found that therapists who have had sexual relations with their their clients are: A. Younger and less experienced as psychotherapists then are those who have not had sexual relations with clients B. More likely than those who have not had sexual relations with clients to have been sexually involved with a supervisor, instructor, or their own therapist C. Likely to say that the sexual relationship have a positive impact on the therapeutic relationship

D. More likely than those who have not had sexual relations with clients to have had other nonsexual dual relationships with clients

D. More likely than those who have not had sexual relations with clients to have had other nonsexual dual relationships with clients (Correct) in their 1998 survey, LAmb and Catanzaro found that psychologist who reported sexual relations with clients were also more likely to report having been involved in nonsexual relationships then were psychologists who had not had sexual relations with clients. Surveys of psychologists have found that those who report sexual relations clients share some characteristics. A. Younger and less experienced as psychotherapists then are those who have not had sexual relations with clients (Incorrect) therapists who become sexually involved with clients tend to be older than those who have not, and professional experience has not been leaked to sexual involvement with clients. B. More likely than those who have not had sexual relations with clients to have been sexually involved with a supervisor, instructor, or their own therapist (Incorrect) D.H. Lamb, S.J. Catanzaro found that there was no statistically significant association between having a sexual relationship with a client and having had a sexual relationship with a supervisor, instructor, or therapist [Psychologists reflect on their sexual relationships with clients, supervisees, and students: Implications for professional practice, professional psychology: research and practice,29(5), 498-503, 1998].

C. Likely to say that the sexual relationship have a positive impact on the therapeutic relationship (Incorrect) in an earlier survey, Lamb and Catanzaro found that psychologist who reported having sexual relations with their clients are more likely to say the relationship have a negative (versus positive) impact [sexual and nonsexual boundary violations involving psychologist, clients, supervisees, and students: implications for professional practice, professional psychology: research and practice, 29(5), 498-503, 1998]

(AATBS Ethics Domain Quiz #18535)

An insurance company requests that you faxed the company confidential client information. Assuming that the client has signed a release for this information, you should: A. Refuse to fax the requested documents B. Make sure the documents are clearly marked as "confidential" C. Make sure that all identifying information is removed or coded

D. Ask the company to verify that the documents have been received after they are faxed

C. Make sure that all identifying information is removed or coded (Correct) you would not have control over who sees the documents once they have been transmitted, and this course of action would ensure that client information is kept confidential. Ethical guidelines that address the confidentiality of client records are relevant to the situation described in this question. See, e.g., Standard 6.02 of the APA's Ethics Code. A. Refuse to fax the requested documents (Incorrect) it would not be necessary to refuse to fax the requested information unless doing so would reach client confidentiality B. Make sure the documents are clearly marked as "confidential"  (Incorrect) marking the materials "confidential"would not guarantee that client confidentiality would be maintained

D. Ask the company to verify that the documents have been received after they are faxed  (Incorrect) this would not necessarily be adequate for ensuring that someone other than an authorized person has had access to the documents

(AATBS Ethics Domain Quiz #18566)

A psychothelapist in private plactice includes the following information on her business card: sliding scale fees, first session free, APA member. This is: A. unethical because of“sliding scale fees.” B. unethical because of“first session free." C. unethical because of“APA member."

D. ethical.

D. ethical (Correct) It is not unethical to charge clients on a sliding scale, and it would only be unethical to provide the first session free if there was some underlying coercion involved. It is also acceptable to list oneself as an APA member as long as it is not done in a way that is misleading. None ofthe information included in the psyi:ho|ogist"s business card is prohibited by ethical guidelines.

(AATBS Ethics Domain Quiz #18547)

Psychologists are most likely to identify which of the following as the most frequently “ethically troubling issue" that they encounter in their professional plactice? A. conduct of colleagues B. confidentiality C. treatment termination

D. sexual issues

B. confidentiality CORRECT Of the 703 ethically troubling issues cited by psychologists in the Pope and Vetter study, the largest number (128) were related to confidentiality. In contrast, conduct of colleagues, treatment termination, and sexual issues were cited, respectively, 29, 5, and 28 times. The issues listed in this answer were all identified as "ethically troubling issues" in a survey of psychologists by K. S. Pope and V.A. Vetter [Ethical dilemmas encountered by members of the American Psychological Association: A national survey, American Psychoiogist, 47(3). 397-411, 1992](AATBS Ethics Domain Quiz #18547)

A client Dr. Amanda Alto has been seeing in therapy for three months is suddenly fired from her job and can no longer afford Dr. Alto's fee. The client suggests that she babysit for Dr. Alto's children in exchange for therapy. In terms of ethical guidelines, this exchange would be: A. ethical. B. ethical only if Dr. Alto discusses the potential for conflict with the client. C. ethical only if a “fair market value" is established forthe client's babysitting services.

D. unethical.

D. unethical CORRECT The situation described in this question violates ethical guidelines regarding multiple relationships and therapy fees. With regard to multiple relationships, if Dr. Alto agrees to the arrangement, she will be acting as both therapist and employer, which may impair her ability to provide effective therapy to the client. In addition, the arrangement represents a form of barter (which is explicitly addressed by Standard 6.05 of the APA's Ethics Code and indirectly addressed by Principles II.1 and 2 ofthe Canadian Code of Ethics] that could also interfere with her objectivity. The situation described in this question is addressed by ethical guidelines related to multiple relationships and therapy fees.

(AATBS Ethics Domain Quiz #18515)

Sally S., age 14, tells her therapist that she want to die and that she has access to her mother’s sleeping pills. The therapist believes that Sally's threat is serious and tells Sally that he'll have to contact  her parents. Sally says she thought they had agreed when she started therapy that everything she told the therapist would be confidential. If the therapist contacts Sally’s parents, he has acted: A.  unethically if Sally was, in fact, told that confidentiality would be maintained. B. unethically if he contacts Sally's parents without first obtaining Sallys consent to do so. C. ethically as long as the limits of confidentiality were discussed with Sally at the outset of therapy.

D. ethically since a breach of confidentiality is justified when a client is suicidal.

D. ethically since a breach of confidentiality is justified when a client is suicidal CORRECT As noted above, psychologists must take appropriate action whenever a client is suicidal. Because Sally is a minor, contacting her parents would be an appropriate action. Exceptions to confidentiality and privilege are addressed in the Ethics Manual of the written study materials. Psychologists are legally and ethically obligated to breach confidentiality whenever a client is believed to be a danger to him/herself.

(AATBS Ethics Domain Quiz #18474)

A client whom Dr. Oliver Olivetti has been seeing for several months has recently changed jobs, and the client has learned that her new insurance plan does not begin covering pre-existing conditions for 12 months. The client asks Dr. Olivetti if he could write his bills so that it appears that she has just started therapy. If Dr. Olivetti complies with the client's request, he is acting: A. ethically since he is taking the client's welfare into consideration. B. ethically only if not to do so would require the client to quit therapy. C. ethically only if the insurance forms do not require him to sign a statement saying that treatment began after the effective date of the insurance.

D.  unethically.

D.  Unethically CORRECT As noted above, complying with the client's request would represent insurance fraud, which would be both unethical and illegal. See, e.g., Standard 6.04 of the APA‘s Ethics Code.The client in this situation is asking Dr. Olivetti to commit insurance fraud and, if Dr. Olivetti does so, he will be acting both unethically and illegally. A. ethically since he is taking the client's welfare into consideration - Incorrect Although Dr. Olivetti should consider his client's welfare (by, for example, offering a reduced fee so that the client doesn't have to terminate therapy), he would be acting unethically and illegally if he agrees to the arrangement requested by the client. B. ethically only if not to do so would require the client to quit therapy -  Incorrect This arrangement would be illegal and unethical regardless of the ramifications for the client.

C. ethically only if the insurance forms do not require him to sign a statement saying that treatment began after the effective date of the insurance - Incorrect This would still represent insurance fraud since the client and therapist are well aware of the insurance provisions.

(AATBS Ethics Domain Quiz #18435)

You have just started seeing a client who is from a different cultural background than you are from who is having difficulty coping with the recent death of her mother. You have frequentiy worked with clients who are coping with loss, but you have never worked with a client from this background. You'd like to continue seeing this client in therapy, however, because you want to broaden your experience to include clients from diverse cultural backgrounds. Your best course of action would be to: A. continue working with the client but “proceed with caution.” B. seek supervision or consultation and continue to work with the client. C. refer the ciient to another therapist who has experience working with clients from this background.

D. explain the situation to the client and terminate therapy.

B. seek supervision or consultation and continue to work with the client CORRECT This situation is addressed by Standard 2.01(b) of the APA's Ethics Code and Principle II.8 of the Canadian Code of Ethics. Standard 2.01(b) states "where scientific or professional knowledge in the discipline of psychology establishes that an understanding of factors associated with age, gender, gender identity, race, ethnicity, culture, national origin, religion, sexual orientation, disability, language, or socioeconomic status is essential for effective implementation of their services or research, psychologists have or obtain the training, experience, consultation, or supervision necessary to ensure the competence of their services, or they make appropriate referrals." This answer is consistent with this requirement and makes the most sense, given the fact that there is no indication that a referral is necessary [i.e., the question indicates that you have previously worked with clients who have experienced a loss].  It is acceptable (and desirable) for psychologists to acquire new skills and to treat new problems - but only when appropriate supervision or consultation is obtained.

(AATBS Ethics Domain Quiz #18456)

Dr. Molly Mendez, a well—known behavioral psychologist, is asked by her secretary if she would be willing to provide him with therapy to help him overcome a gambling addiction. If Dr. Mendez agrees to do so, she will be acting: A. ethically as long as she has expertise in treating gambling addiction. B. ethically as long as she charges the secretary her usual fee for psychological services. C. unethically since doing so is likely to create an unacceptable multiple relationship.

D. unethically since maintaining the secretary's confidentiality will be difficult in this situation

C. unethically since doing so is likely to create an unacceptable multiple relationship CORRECT A potentially harmful multiple relationship is the primary problem in the situation described in this question. Multiple relationships are addressed in Standard 3.05 of the APA's Ethics Code and Principle III.33 of the Canadian Code of Ethics. These guidelines prohibit multiple relationships that coutd be expected to impair a psychologist‘s objectivity, which is a potential problem in this situation since both roies (employer and therapist) would continue simultaneously over an extended period of time. The situation described in this question represents a multiple relationship - i.e., Dr. Mendez would be acting as both employer and therapist if she agrees to see her secretary in therapy.(AATBS Ethics Domain Quiz #18491)

Which of the foilowing best describes ethical requirements regarding sexual intimacies with supervisees? A.  Ethical guidelines do not prohibit sexual relationships with supewisees. B.  Ethical guidelines prohibit sexual relationships with supervisees under any circumstances. C. Ethical guidelines prohibit sexual relationships with current supervisees only when the relationship will impair the psychologist’s objectivity and effectiveness as a supervisor.

D.  Ethical guidelines prohibit sexual relationships with supervisees over whom the psychologist has evaiuative authority.

D.  Ethical guidelines prohibit sexual relationships with supervisees over whom the psychologist has evaiuative authority - CORRECT Standard 7.07 of the APA's Ethics Code and Principle II.28 of the Canadian Code of Ethics prohibit psychologists from becoming involved in sexual relationships with students and supewisees over whom they have evaluative authority. Being familiar with the exact language of the ethical guidelines would have allowed you to identify the conect response to this question.

(AATBS Ethics Domain Quiz #18556)

A psychologist should be aware that information contained in client records: A. is protected by law and can never be disclosed without the client’s consent. B. is the property of the psychologist and cannot be disclosed without his/her consent. C. is the joint property of the psychologist and client and cannot be disclosed without the consent of both.

D. is not free from disclosure all of the time, regardless of the wishes of the client or the psychologist.

D. is not free from disclosure all of the time, regardless of the wishes of the client or the psychologist - CORRECT Record ownership and access are governed by legal guidelines. In most jurisdictions, the laws grant ownership of the physical record to the practitioner but give clients the right to have access to the information contained in the record. In addition, there are times when other individuals or entities have the right to access client information
(e.g., confidentiality of client information is waived in certain situations in the military and privilege is waived in certain legally defined situations). Therefore, this is the best answer  of those given.There are some situaljons when client infomiation must be released, despite the wishes of the client or the psychologist.(AATBS Ethics Domain Quiz #18487)

A psychology intern who has been working at a mental health clinic for the past six months tells you that his internship was suddenly terminated by his supervisor for “incompetence.” He also says that he had not been given any warning that his performance was inadequate before being terminated. You tell the intern that:  A. the supervisor's behavior seems unfair but is not addressed by ethical guidelines. B. the supervisor's behavior is acceptable as long as the intern was given specific reasons for his termination. C.  the supervisor's behavior is acceptable as long as the reason for the intern's termination is valid.

D.  the supervisor's behavior may have violated ethical guidelines regarding supervisee performance evaiuations.

D.  the supervisor's behavior may have violated ethical guidelines regarding supervisee performance evaiuations - CORRECT This answer is most consistent with ethical guidelines regarding evaluation and feedback as well as with more general provisions regarding the avoidance of harm (e.g., Standard 3.04 of the Ethics Code and Principles 1.13 and 11.2 of the Canadian Code of Ethics). Although termination of supervisees is not directly addressed by ethical guidelines, evaluating and providing feedback to supervisees is addressed by Standard 7.06 of the APA's Ethics Code and Principle ll.25 of the Canadian Code of Ethics.

(AATBS Ethics Domain Quiz #18517)

Which of the following best describes the requirements of the privacy rule of the Health Insurance Portability and Accountability Act (HIPAA)? A. The privacy rule always takes precedence over state laws. B. The privacy rule is superceded by state laws when those laws provide greater privacy protection. C.  The privacy rule is enforceable only in the absence of state and other relevant federal laws

D. The privacy rule applies only to organizations that receive financial assistance from the federal government.

B. The privacy rule is superceded by state laws when those laws provide greater privacy protection - CORRECT The privacy rule provides minimum standards for access to and use of patient information and is superceded by other laws that provide patients with greater control over their health records. HlPAA‘s privacy rule provides standards for protecting the privacy and security of health information that is transmitted electronically.
 (AATBS Ethics Domain Quiz #18561)

During a discussion with a‭ ‬4th grade teacher,‭ ‬the teacher tells a school psychologist that one of her students is constantly disrupting the class.‭ ‬After evaluating the child and obtaining more information about the situation,‭ ‬the psychologist concludes that the teacher's approach with the child is part of the problem.‭ ‬Consequently,‭ ‬the psychologist should:‬‬‬‬‬‬‬‬‬‬‬‬‬‬‬‬‬‬  a. make an appointment with the principal to discuss the teacher’s approach.   b. explain to the teacher how her approach may be exacerbating the problem.  c. recommend that he‭ (‬the therapist‭) ‬start counseling the child on a regular basis. ‬‬‬‬‬‬

 d. recommend that the child be transferred to another classroom

  b. explain to the teacher how her approach may be exacerbating the problem. CORRECT General Principle B of the APA’s Ethics Code states that "psychologists consult with, refer to, or cooperate with other professionals and institutions to the extent needed to serve the best interests of those with whom they work."  School psychologists must often act as a coordinator to bring together a variety of people and resources to resolve problems. Of the answers given, this one is most in line with the child's best interests and the school psychologist's responsibility to work with all people involved in the case. Although this situation is not explicitly addressed by ethical guidelines, the APA’s Ethics Code and the Canadian Code of Ethics require psychologists to cooperate with other professionals.

(AATBS Ethics Domain Quiz #18561)

At the end of her third therapy session with Dr. Beatrice Bramwell, a client reveals that he is also in therapy with another psychologist. As an ethical psychologist, Dr. Bramwell should: A.  inform the client that he'll have to make a choice in the near future about which therapist to continue seeing. B. inform the client that it would be unethical to continue seeing him while he is receiving treatment from another psychologist. C. call the other psychologist to inform him of the situation and obtain his permission to continue therapy with the client.

D. discuss the treatment issues relevant to this situation with the client during the next session.

D. discuss the treatment issues relevant to this situation with the client during the next session - CORRECT In this situation, Dr. Bramwell would want to consider the client's welfare and avoid duplicating services or working at cross purposes with the other psychologist. Consequently, of the answers given, this is the best one. This issue is explicitly addressed by Standard 10.04 01 the APA‘s Ethics Code and indirectly addressed by Principle ll.18 of the Canadian Code of Ethics.
(AATBS Ethics Domain Quiz #18563)

In a research study in which it is necessary to use deception, participants are not told the exact nature of the study prior to their participation. In terms of ethical requirements, in this situation the investigator is obligated to: A. explain why participants were not informed in the presentation of his research findings. B. explain the true nature of the study to all participants as soon as possible. C. file a waiver of informed consent with the institutional review board prior to beginning the study.

D. do none of the above as long as the study is determined to be of significant scientific value

B. explain the true nature of the study to all participants as soon as possible - CORRECT When deception is necessary, participants must be provided with an explanation of the purpose of the study as soon as possible.The use of deception is not prohibited by ethical guidelines but, when it is used, it must be accompanied by certain actions. Guidelines for the use of deception are provided in Standard 8.0? of the APA's Ethics Code and are discussed in the Ethics Manual in the written study matenals. A. explain why participants were not informed in the presentation of his research findings - Incorrect This is not required by the ethical guidelines. C. file a waiver of informed consent with the institutional review board prior to beginning the study - Incorrect This is not required by ethical guidelines. D. do none of the above as long as the study is determined to be of significant scientific value - Incorrect This is incorrect since "significant value“ does not preclude explaining the true

purpose of the study to participants as soon as possible.

(AATBS Ethics Domain Quiz #18543)

Which of the following best describes ethical guidelines regarding the use of inducements to encourage individuals to participate in a research study? A. Financial and other inducements are acceptable only when they do not coerce participation in the research study. B. Financial and other inducements are acceptable only when the risks for participating in the study are ciearly described to potential participants. C. Financial and other inducements are acceptable only when alternative methods for obtaining participants are unavailable.

D. Financial inducements are unacceptable under any circumstances.

A. Financial and other inducements are acceptable only when they do not coerce participation in the research study - CORRECT This answer is most consistent with ethical guidelines. For example, Standard 8.06 of the Ethics Code states that "psychologists make reasonable efforts to avoid offering excessive or inappropriate financial or other inducements for research participation when such inducements are likely to coerce participation." The use of inducements to encourage participation in a research study is addressed by Standard 8.06 of the APA's Ethics Code and Piinciples I27 and III.32 of the Canadian Code of Ethics. B. Financial and other inducements are acceptable only when the risks for participating in the study are ciearly described to potential participants -  Incorrect This answer does not accurately describe ethical guidelines regarding the use of inducements. C. Financial and other inducements are acceptable only when alternative methods for obtaining participants are unavailable - Incorrect This answer does not accurately describe ethical guidelines regarding the use of inducements.

D. Financial inducements are unacceptable under any circumstances - Incorrect Financial and other inducements are acceptable in some circumstances.

(AATBS Ethics Domain Quiz #18558)

You receive a request from a psychologist working at a mental health crisis unit for information about a former client of yours who ieft three months ago with an unpaid bill of over $500. The client has been admitted as an inpatient and has signed a release. You tell the psychologist that you will provide her with the information she has requested only after the client has made arrangements for paying you the money he owes. You have: A. acted ethically as long as you had informed the client of your policy regarding unpaid fees. B. acted ethically since the ciient has violated your implicit contract with him to pay the fee for the services you provided to him. C. acted ethically since this action is not covered by the ethical guidelines.

D. acted unethically.

D. acted unethically - CORRECT The ethical guidelines prohibit "holding a client"s records hostage" because
the client has not paid his or her bill when these records are requested and needed for emergency treatment. See Standard 6.03 of the APA's Ethics Code which explicitly prohibits this practice and Principles II.1 and 2 of the Canadian Code of Ethics which require psychologists to promote the welfare of clients and to avoid harming them. In this situation, the fomler client's records are needed for an emergency situation (i.e., the client has been admitted as an inpatient to a mental health crisis unit).

(AATBS Ethics Domain Quiz #18469)

At the beginning of therapy with Mark M., you discussed the possible duration of treatment and estimated that therapy would not extend beyond his insurance coverage of your fee. You also informed Mark that you would be unable to reduce your fee when his therapy was no longer covered by insurance. Unfortunately, that time has arrived: Mark's insurance will cover only the next three sessions, many of Mark's problems are still unresolved, and he cannot afford your full fee. In terms of ethical responsibilities, you are: A. required to see Mark at a reduced or deferred fee until his problems are adequately resolved. B. required to see Mark at a reduced or deferred fee until he feels ready to terminate treatment. C. required to discuss the matter with Mark and make a referral at the end of the three sessions if necessary.

D. not required to continue seeing Mark since you discussed this matter at the beginning of therapy.

(C. required to discuss the matter with Mark and make a referral at the end of the three sessions if necessary - CORRECT You are not required to continue seeing a client if the client cannot pay your fee. However, to avoid abandoning the client when insurance coverage has ended, you should either offer services at a reduced or deferred fee or, if that is not feasible, make appropriate referrals. Standard 6.04 of the APA's Ethics Code and Principle ll.32 of the Canadian Code of Ethics apply to this situation.
D. not required to continue seeing Mark since you discussed this matter at the beginning of therapy -  Incorrect Discussion of this issue at the beginning of therapy is a good practice but does not eliminate your responsibility to the client when his insurance runs out.

(AATBS Ethics Domain Quiz #18431)

Dr. Mansfield Mayhem receives a subpoena requiring him to testify about a current therapy client at a trial. Dr. Mayhem contacts the client who states that she does not want him to release any confidential information to the court. Dr. Mayhem should: A. notify the attorney who issued the subpoena that the client has asserted the privilege and request that he be released from subpoena. B.  do nothing further until he receives an order from the court to appear at the trial. C.  notify the court that he will not be appearing at the trial because the client does not want him to do so.

D.  advise the attorney who issued the subpoena that he cannot appear as requested because the client has not given him permission to do so.

A. notify the attorney who issued the subpoena that the client has asserted the privilege and request that he be released from subpoena. CORRECT When the client invokes the privilege in this type of situation, the psychologist should contact the client's attorney or his,/her own attorney to discuss the matter. The psychologist can also contact the attorney who issued the subpoena to request to be released from it. Unless the psychologist receives a written release, he she must appear as requested but would not reveal confidential (privileged) information without the client's consent or without an order from the court to do so. A subpoena is a legal document that requires a psychologist to appear at a designated time at a legal proceeding. If the client invokes the privilege (says she does not want the therapist to release confidential information to the court), the subpoena is still valid and the psychologist must appear unless he/she has been released from it.

(AATBS Ethics Domain Quiz #18459)

You are contacted by a defendant's attorney to evaluate the defendant to determine if there are grounds for the insanity defense. Who is the "holder of the privilege" in this situation? A. the defendant B. the defendant's attorney C. the court

D. you (the psychologist)

A. the defendant - CORRECT The defendant would still be the "holder of the privilege" even if privilege is waived. (Note that, in the situation described in this question, privilege would be waived only if the defendant actually claims insanity as hisiher defense during the trial.) Additional information on privilege is provided in the Ethics Manual in the written study materials. Keep in mind that privilege is waived only in certain circumstances and that, even when it is waived, another person or agency does not become the "holder of the privilege."

(AATBS Ethics Domain Quiz #18527)

Which of the following is not a condition for a claim of malpractice against a psychologist? A. The psychologist must have a legal duty to the client. B. There must be evidence that the psycho|ogist's actions were not in the best interests of the client. C. The psychologist must have acted in a negligent or otherwise improper manner.

D. There must be a causal relationship between the psycho|ogist's negligence and the harm claimed by the client.

B. There must be evidence that the psycho|ogist's actions were not in the best interests of the client. - CORRECT This does not accurately describe one of the four conditions. A claim of malpractice requires that four conditions be met: a duty to the client; a breach of that duty, harm to the patient; and a causal relationship between the breach of duty and harm to the patient. A. The psychologist must have a legal duty to the client. - Incorrect This is one of the four conditions for a malpractice claim. C. The psychologist must have acted in a negligent or otherwise improper manner - Incorrect This also describes a condition for a claim of malpractice.

D. There must be a causal relationship between the psycho|ogist's negligence and the harm claimed by the client - . Incorrect This is one of the four conditions of malpractice.

(AATBS Ethics Domain Quiz #18519)

Which of the following is not an element of informed consent? A. credibility B. comprehension C. voluntariness

D. competence

A. credibility - CORRECT In order for a consent to be informed, the individual must have adequate information (knowledge) and be able to comprehend that information, must give consent voluntarily (without coercion), and must have the competence (capacity) to make rational decisions. Credibility is not one of the conditions for consent. Additional information on informed consent is provided in the Ethics Manual in the written study materials. Legally, three conditions are required for a consent to be tmiy "informed." Note that this question is asking for the element that is NOT required.

(AATBS Ethics Domain Quiz #18540)

Alice A.,‭ ‬who has a master's degree in clinical psychology but is not licensed,‭ ‬is hired by a community mental health clinic to,‭ ‬among other things,‭ ‬administer several standard psychological tests to adult clients.‭ ‬According to ethical guidelines,‭ ‬this is:‬‬‬‬‬‬‬‬‬‬‬‬‬‬‬‬‬‬ a. acceptable as long as a professional clinical psychologist co-signs‭ ‬Alice’s evaluations. ‬‬‬ b. acceptable as long as a professional clinical psychologist supervises‭ ‬Alice’s activities. c. unacceptable since‭ ‬Alice is not a licensed psychologist. ‬‬‬

d. unacceptable since ‬Alice does not have a doctoral degree. 

b. acceptable as long as a professional clinical psychologist supervises‭ ‬Alice’s activities. (Correct Answer)‬‬‬ This answer is most consistent with ethical and legal guidelines. Alice may administer some psychological tests as long as she has been adequately trained and is supervised by a professional clinical psychologist. Ethical (and legal) guidelines do not prohibit the provision of all psychological services by individuals who are not licensed clinical psychologists.  However, they do require that such persons be supervised by a professional clinical psychologist.
a. acceptable as long as a professional clinical psychologist co-signs‭ ‬Alice’s evaluations. - Incorrect This would not be considered adequate supervision.

(AATBS Ethics Domain Quiz #18450)

Which of the following best describes ethical guidelines regarding barter as a substitute for the collection of fees for professional psychological services‭?‬‬‬ a. Barter may be acceptable when certain conditions are met.  b. The barter of goods‭ (‬but not services‭) ‬is prohibited by ethical guidelines. ‬‬‬‬‬‬ c. The barter of services‭ (‬but not goods‭) ‬is prohibited by ethical guidelines. ‬‬‬‬‬‬

d. Barter of any kind is prohibited by ethical guidelines.

a. Barter may be acceptable when certain conditions are met. (CORRECT) Barter is prohibited by Standard 6.05 of the APA’s Ethics Code when it is clinically contradindicated or exploitative – in other words, in certain circumstances, barter may be acceptable. The Canadian Code of Ethics does not directly address barter but, to be consistent with the “spirit” of the Code, barter should be avoided when it may be harmful to the client. Barter is explicitly addressed by Standard 6.05 of the APA’s Ethics Code and indirectly addressed by Principles III.31-33 of the Canadian Code of Ethics.

(AATBS Ethics Domain Quiz #18518)

Dr. Melba Morrison, a licensed psychologist, conducts research and teaches several classes at a large university. In her social psychology classes, she usually covers only material that is consistent with the results of her own research. In terms of the ethical guidelines, this practice is: A. ethical since it is up to Dr. Morrison to decide what to cover in her classes. B. ethical as long as Dr. Morrison provides information about her policy in the class syllabus C. ethically questionable but not addressed by ethical guidelines.

D. clearly unethical.

D. clearly unethical - CORRECT To be consistent with ethical requirements when acting as an instructor or trainer, psychologists must present information accurately and in an unbiased way. Therefore, Dr. Morrison is acting unethically by presenting only information that is consistent with the results of her own research. This issue is addressed by Standard 7.03 of the APA's Ethics Code and Principle III.12 of the Canadian Code of Ethics.

(AATBS Ethics Domain Quiz #18562)

A 24-year-old woman calls Dr. Sylvia Silby for an appointment but refuses to discuss her problem on the phone. During the first therapy session, Dr. Silby finds the client to be an attractive woman, dressed in a somewhat flashy manner. The young woman discusses her difficulties in a vague, guarded manner, indicating that, for the past few months, she has had increasing difficulty sleeping and has felt anxious and irritable. Ten minutes before the end of the session, the young woman tells Dr. Silby that she has been seeing another therapist for six months and that, about four weeks ago, she started having a sexual relationship with him. As an ethical psychologist, Dr. Silby should: A. advise the woman to terminate treatment with the other psychologist and begin therapy with her. B. explain to her the seriousness of her allegations and advise her of her rights and alternative courses of action. C. contact the other therapist to discuss the matter with him.

D. file a complaint against the other therapist with the ethics committee and recommend to the woman that she immediately terminate treatment with him.

B. explain to her the seriousness of her allegations and advise her of her rights and alternative courses of action - CORRECT Dr. Silby cannot contact the therapist or make a report herself (answers c and d) without the client's consent since to do so would represent a breach of confidentiality. Therefore, of the actions listed in the answers, the best one is to discuss the matter with the client and advise her of her options. If the woman's allegations are true, the original therapist has violated ethical guidelines and may have violated the law. Ethical violations by colleagues are covered in Standards 1.04 and 1.05 of the APA‘s Ethics Code and are discussed in the Ethics Manual of the written study materials.

(AATBS Ethics Domain Quiz #18453)

You believe that one of your clients is exhibiting symptoms of PTSD. A new test for PTSD has been developed by a psychologist at the local university, and you would like to use the test to evaluate the client. However, the test is currently labeled “For Research Purposes Only." You should: A. get permission to use the test from the psychologist who developed it. B. get an informed consent from the client before administering the test. C. make sure that you combine test results with other data you have collected.

D. not use the test.

D. not use the test - CORRECT "For research purposes only" implies that this test has not been validated for
clinical use and, therefore, should not be used for that purpose. Ethical guidelines require that tests be used only for the purposes for which they have been validated.

(AATBS Ethics Domain Quiz #18539)

Dr.‭ ‬Ja-Hoon Jang is conducting a research study that involves interviewing junior high school students about their relationships with peers.‭ ‬Prior to beginning the study,‭ ‬Dr.‭ ‬Jang obtained informed consents from the parents of the students as well as assents from the students themselves.‭ ‬During the course of the study,‭ ‬one of the students says he doesn’t want to complete the interview.‭ ‬Dr.‭ ‬Jang should: A. tell the student that he cannot withdraw from the study since one of his parents has signed an informed consent.   B. tell the student that he cannot withdraw from the study since he (‬the student) ‬agreed to participate at the beginning of the study.   C. contact the student’s parents to ask them to encourage their son to complete the interview.  

D. allow the student to withdraw from the study.

 D. allow the student to withdraw from the study - (CORRECT) Ethical guidelines require researchers to allow participants to withdraw at any time from a research study. This answer is most consistent with this requirement. This issue is addressed by Standard 8.02 of the APA’s Ethics Code and Principle I.24 of the Canadian Code of Ethics.

(AATBS Ethics Domain Quiz #18554)

Dr. Kay Klein, a licensed psychologist, is preparing a brocure for her weekend workshop on "Effective Parenting Strategies for Single Parents." Included in the brocure are several testimonials from former workshop participants. According to ethical guidelines, this use of testimonials is: A. Clearly ethical B. Ethical if certain conditions are met C. Unethical only when the testimonials are false or misleading

D. Clearly unethical

B. Ethical if certain conditions are met - CORRECT Standard 5.05 states that psychologists do not solicit testimonials “from current therapy clients! patients or other persons who because of their particular circumstances are vulnerable to undue infiuence"; and Principle III.31 prohibits psychologists from exploiting clients. Therefore, of the answers given, this one is most consistent with ethical guidelines.The use of testimonials is addressed by Standard 5.05 of the APA's Ethics Code and Principle lll.31 of the Canadian Code of Ethics.
C. Unethical only when the testimonials are false or misleading - Incorrect Although false or misleading testimonials would be unethical, this answer is not the best one because it states that testimonials are unethical only when they contain false or misleading information.

(AATBS Ethics Domain Quiz #18521)

"Privilege": A. is an ethical requirement designed to protect client information from being disclosed except when the client has signed a release. B.  is a legal requirement designed to protect confidential information from being disclosed to anyone not directly involved in a client's case. C. is an ethical requirement that applies to the confidentiality of therapist—client communications in both legal and nonlegal situations.

D. is a legal requirement designed to protect confidential information from being disclosed in a legal proceeding.

D. is a legal requirement designed to protect confidential information from being disclosed in a legal proceeding - CORRECT Privilege is “a legal term describing certain specific types of relationships that enjoy protection from disclosure in legal proceedings" [G. P., Koocher, Privacy, confidentiality, and privilege, in G. P. Koocher, J. C. Norcross &S. S. Hill (eds), Psychologists’ Desk Reference, Oxford University Press, New York, 1998]. Privilege is discussed in the Ethics Manual of the written study materials. Privilege is a legal term and refers to "privileged communications" that arise in legally designated relationships (e.g., the therapist-client relationship).

(AATBS Ethics Domain Quiz #18462)

A psychologist is the supervisor of several interns at a community hospital in a small town and notices that one of the interns seems unenthusiastic and depressed.‭ ‬When he talks to her,‭ ‬she tells him that she’s been very unhappy lately and feels she has no one to turn to.‭ ‬The intern asks the psychologist to see her in therapy,‭ ‬and he agrees to do so.‭ ‬The psychologist has acted: A. unethically by agreeing to get involved in a dual relationship.   B. unethically by violating the intern''s right to privacy.   C. ethically because they are working in a community hospital in a small town.  

D. ethically because the intern is in desperate need of help.

A. unethically by agreeing to get involved in a dual relationship -  (CORRECT) Standard 3.05(a) of the Ethics Code states that “a psychologist refrains from entering into a multiple relationship if the multiple relationship could reasonably be expected to impair the psychologist's objectivity, competence, or effectiveness in performing his or her functions as a psychologist, or otherwise risks exploitation or harm to the person with whom the professional relationship exists.” The psychologist in the situation described in this question is entering into a multiple relationship that may threaten his objectivity and effectiveness as a supervisor and therapist. In most situations, psychologists should avoid dual (multiple) relationships. See, e.g., Standard 3.05(a) of APA's Ethics Code and Principle III.33 of the Canadian Code of Ethics. B. unethically by violating the intern''s right to privacy -  Incorrect There is no indication that privacy is an issue in this situation. C. ethically because they are working in a community hospital in a small town -  Incorrect In some circumstances, multiple relationships may be unavoidable. However, the fact that the intern and psychologist work in a community hospital in a small town does not necessarily mean that no other options are available.

D. ethically because the intern is in desperate need of help - Incorrect There is no indication that the intern is in such “desperate need of help” that she cannot wait to see another professional.

(AATBS Ethics Domain Quiz #18445)

To serve as an expert witness in a court case,‭ ‬a psychologist must:‬‬ a. be board certified as an expert witness.  b. be qualified by the court to offer opinions and conclusions about the case. c. have been hired by the person he/she will be testifying about. 

d. testify only about what he/she personally observed or obtained.

b. be qualified by the court to offer opinions and conclusions about the case. (CORRECT) Expert witnesses are qualified by the court to offer opinions and conclusions. An expert witness is a person who, because of his/her special training, knowledge, or experience, is qualified to offer an opinion as testimony in a court proceeding. 

(AATBS Ethics Domain Quiz #18473)

Dr. Calvin Claymore, a licensed Psychologist, regularly waives the co-payment for low-income clients whose therapy fees are covered by insurance. This practice is:  A. ethical only if the insurance company has agreed to this arrangement. B. ethical since it is the best interest of low-income clients C. ethical as long as Dr. Claymore doesn't bill the insurance company for an increased hourly fee in order to collect her full fee.

D. ethical as long as the insurance company does not explicity prohibit this arrangement.

A. ethical only if the insurance company has agreed to this arrangement. - CORRECT The practice described in this question represents insurance fraud, which is both unethical and illegal. Most insurance companies agree to pay a specific percentage of a psychologist's fee but, when the co-payment is waived, this means the insurance company is paying the entire fee. Consequently, regularly waiving the co-payment without the permission of the insurance company represents insurance fraud. B. ethical since it is the best interest of low-income clients - Incorrect C. ethical as long as Dr. Claymore doesn't bill the insurance company for an increased hourly fee in order to collect her full fee. - Incorrect

D. ethical as long as the insurance company does not explicity prohibit this arrangement. - Incorrect

(AATBS Ethics Domain Quiz #18479)

Within the context of sexual harassment,‭ “‬quid pro quo‭” ‬refers to:‬‬‬‬ a. a situation in which an employee’s response to sexual demands impacts his/her job. b. the creation of a‭ "‬hostile environment‭" ‬as a result of sexually offensive conduct. ‬‬‬‬ c. the judgment of a‭ "‬reasonable woman‭" ‬that an act is offensive. ‬‬‬‬

d. overt‭ (‬explicit‭) ‬sexual conduct.‬‬‬‬

a. a situation in which an employee’s response to sexual demands impacts his/her job. (CORRECT) Quid pro quo sexual harassment occurs when a person’s submission to or rejection of sexual advances affects his/her employment status. (Hostile environment sexual harassment is the other legally recognized type of sexual harassment.) “Quid pro quo” is one of two legally recognized types of sexual harassment.

(AATBS Ethics Domain Quiz #18536)

A psychologist decides to use a computerized test service to facilitate scoring and interpretation of the MMPI-2,‭ ‬which he frequently administers to his clients. ‭ ‬The psychologist should be aware that: A. the use of computerized interpretations is prohibited by ethical guidelines.   B. computerized scoring and interpretation services are notoriously unreliable.   C. computerized interpretations should always be supplemented with other information obtained by the psychologist.  

D. because of their objectivity,‭ ‬computerized interpretations are preferable to subjective interpretations.

C. computerized interpretations should always be supplemented with other information obtained by the psychologist - (CORRECT) The information in a computerized interpretation is limited and, therefore, the best policy is to always supplement computerized interpretations with information from other tests, interviews, observations, etc.  Automated scoring and interpretation services are available for a number of psychological tests including the MMPI-2. A. the use of computerized interpretations is prohibited by ethical guidelines -  Incorrect The use of automated interpretation services is not prohibited by ethical guidelines as long as certain standards are met by both the service and the users of the service. B. computerized scoring and interpretation services are notoriously unreliable - Incorrect The quality of computer interpretations varies from service to service, and it cannot be concluded that all computerized interpretations are "notoriously unreliable."

D. because of their objectivity,‭ ‬computerized interpretations are preferable to subjective interpretations - Incorrect The information provided by computerized interpretations is limited and the accuracy of the interpretation depends on the quality of the service. Therefore, it cannot be concluded that computerized interpretations are necessarily the preferred method of score interpretation. 

(AATBS Ethics Domain Quiz #18447)

A psychologist receives a phone call from a person whom she had an intimate (including sexual) relationship with for a short period of time over two years ago. The relationship ended amicably and this person wants to start therapy with the psychologist. With regard to ethical requirements, which of the following is true about this situation? A. It would be ethical to begin therapy with this person as long as the psychologist does not revive their previous personal relationship. B. It would be ethical to begin therapy with this person since two years have passed since the end of the relationship. C. It would be ethical to begin therapy with this person as long as the psychologist discusses the potential for conflicts during the informed consent process.

D. It would not be ethical to begin therapy with this person.

D. It would not be ethical to begin therapy with this person - CORRECT This answer is most consistent with Standard 10.07, which explicitly prohibits psychologists to accept former sexual partners as therapy clients and Principle III.33, which prohibits multiple relationships that might impair a psychologist's objectivity. This situation is directly addressed in Standard 10.0? of the APA‘s Ethics Code and indirectly addressed by Principle III.33 ofthe Canadian Code of Ethics.

(AATBS Ethics Domain Quiz #18485)

A ___________ is a person appointed by the court to represent and make decisions in a legal proceeding for a minor or other person legally incapable of doing so. A. testamentary guardian B. guardian ad litem C. conservator of a minor

D. special conservator

B. guardian ad litem - CORRECT The description presented in this question best describes a guardian ad litem who is appointed by the court for a specific purpose (e.g., to make decisions for someone who is incapable of doing so). For the licensing exam, you‘ll want to be familiar with the item that is being asked about by this question. Additional infomiation about guardianship is provided in the Ethics Manual of the written study materials. A. testamentary guardian - Incorrect A testamentary guardian is a person who is appointed in a deed or will to serve as guardian of a minor or disabled person. C. conservator of a minor - Incorrect A conservator of a minor is responsible for managing the minor‘s finances.

D. special conservator - Incorrect A special conservator carries out the terms of a preliminary protective order.

(AATBS Ethics Domain Quiz #18500)

A psychologist is writing a book for popular distribution and illustrates the section on projective personality tests with a number of the pictures used in the Hand Test. This use of test materials in this way is: A. clearly unethical because it compromises the usefulness of the test. B. ethical only if the publisher of the book has accepted responsibility for including the pictures. C. ethical since the HAnd Test is not a commonly used psychological test.

D. Professionally questionable but not covered by ethical guidelines.

A. clearly unethical because it compromises the usefulness of the test. - CORRECT This question addresses the issue of test security. This answere is most consistent with the provisions of Standard 9.11 of the APA's Ethics Codes and PRinciples IV.11 of the Canadian Code of Ethics. Standard 9.11 states that "psychologists make reasonable efforts to maintain the integrity and security of test materials and other assessment techniques"; and Principles IV.11 states that psychologists "protect the skills, knowledge, and interpretations of psychology from being misused, used incompetently, or made useless (e.g., loss of security of assessment techniques) by others." B. ethical only if the publisher of the book has accepted responsibility for including the pictures. - Incorrect C. ethical since the HAnd Test is not a commonly used psychological test. - Incorrect

D. Professionally questionable but not covered by ethical guidelines. - Incorrect

(AATBS Ethics Domain Quiz #18493)

Malpractice is a tort involving which of the following? A. Failure to protect B. Coercion C. Negligence

D. Breach of Contract

C. Negligence - CORRECT Malpractice is a civil tort involving a negligent breach of duty that results in harm to the client. Being familiar with the conditions for a claim of malpractice would have helped you identify this as the correct answer (see the Ethics Manual in the written study materials). Atort is a civil wrongdoing resulting from a breach of duty. A. Failure to protect - Incorrect Although a tort involves a breach of duty, it is not necessarily related to the duty to protect (although a failure to protect in certain legally defined situations might

result in a claim of malpractice).

(AATBS Ethics Domain Quiz #18522)

You have been seeing Malcolm M. for several months and believe that the original issues that Malcom came to therapy for have been successfully resolved. When you bring up the topic of termination with Malcolm, he says that he wants to continue therapy. In subsequent sessions, you and Malcolm are unable to come up with additional treatment goals. As an ethical psychologist, your best course of action would be to: A. tell Maicolm that you are ethically required to terminate therapy and reassure him that he'll be okay. B. provide Malcolm with a brief period of pretermination counseling and discuss the reasons why termination is appropriate. C. establish a contract with Malcolm for a maximum of three more months of therapy.

D. continue seeing Malcolm until he feels he is ready to stop seeing you.

B. provide Malcolm with a brief period of pretermination counseling and discuss the reasons why termination is appropriate - CORRECT Of the responses given, this is most consistent with ethical guidelines. Standard 10.10 of the APA‘s Ethics Code requires psychologists to terminate therapy when it is “ reasonably clear that the client/patient no longer needs the service" but to do so only after providing the appropriate pretennination counseling. Principles III.31 and 37 of the Canadian Code of Ethics contain similar language. Ethical guidelines are very clear about situations in which a therapist believes that it is the appropriate time to terminate therapy.

(AATBS Ethics Domain Quiz #18537)

You receive an e-mail from Dr. Brenda Browne, a licensed psychologist, who is currently seeing one of your former clients for a substance use disorder. Dr. Browne asks you to forward the client's file to her and states that the client has signed a release authorizing you to do so. You should: A. wait until you receive a request from the client before taking any action. B. contact the client to discuss the release of information. C. forward a photocopy of the file to the psychologist as requested.

D. forward a summary of the file to the psychologist that includes only information related to the client's current condition.

B. contact the client to discuss the release of information - CORRECT Dr. Browne would want to verify that the client has, in fact, signed a release and discuss the release and its potential consequences with the client before providing the psychologist with the requested information. Even though the person requesting the client's record is a licensed psychologist. Dr. Browne would not want to breach confidentiality by providing confidential client  infomation to her without knowing specifically what the client has consented to.

(AATBS Ethics Domain Quiz #18461)

A client is in therapy with you as part of the requirements for her probation. During the third therapy session, she tells you that she wants to quit therapy. You should: A. tell her that she cannot do so. B. tell her that she can quit but that she needs to consider the legal ramifications of doing so. C. tell her that her desire to quit is normal resistance and that she should come to therapy for at least two more sessions before making a decision.

D. call the police immediately.

B. tell her that she can quit but that she needs to consider the legal ramifications of doing so - CORRECT This answer is most consistent with Standard 3.07 and Principle I.26, which address a psychologist's responsibilities when providing services at the request of a third party. In this situation, the psychologist must ensure that the client understands what legal sanctions she may face if she does not stay in therapy. The situation described in this question is addressed by Standard 3.07 of the APA‘s Ethics Code and Principle l.26 of the Canadian Code of Ethics.

(AATBS Ethics Domain Quiz #18499)

A psycho|ogist's behavior as a private citizen: A. can never serve as grounds for a charge of unethical conduct. B. may serve as grounds for a charge of unethical conduct if it violates ethical guidelines C. may serve as grounds for a charge of unethical conduct if it compromises his/her professional responsibilities.

D. will serve as grounds for a charge of unethical conduct when it also violates the law.

C. may serve as grounds for a charge of unethical conduct if it compromises his/her professional responsibilities - CORRECT The Introduction to the APA's Ethics Code states that the "purely private conduct of psychologists  is not within the purview of the Ethics Code." However, Principle C (Professional and Scientific Responsibility) notes that when "psychologists' conduct may compromise their professional responsibilities or reduce the public's trust in psychology and psychologists," personal misconduct may constitute a violation of the Ethics Code. Similar language is included in the Preamble of the Canadian Code of Ethics. A psychologists private behaviors are addressed in the Introduction and Principle C of the APA's Ethics Code and the Preamble of the Canadian Code of Ethics.
D. will serve as grounds for a charge of unethical conduct when it also violates the law - Incorrect A minor legal infraction (e.g., traffic violation) is not likely to be grounds for a charge of unethical conduct.

(AATBS Ethics Domain Quiz #18568)